V3 block 2

¡Supera tus tareas y exámenes ahora con Quizwiz!

A 42-year-old man presents with a 10-day history of worsening headache, stuffy nose, greenish nasal discharge, and a low grade fever. He has body aches and facial pain, as well as a dry cough. He denies shortness of breath, abdominal pain, nausea, or vomiting. He is a non-smoker, has no significant past medical history, and is only taking acetaminophen. On exam, he has a temperature of 100.9°F taken orally. Pulse is 86/min, BP is 120/76 mm Hg left arm sitting, and SPO2 is 94% on room air. Lungs are clear and abdomen normal. Nasal mucosa appears boggy, and there is tenderness with palpation over the facial bones (maxillary area). Pharynx is without exudates. Question What component of the history is the most informs your decision to prescribe antibiotics for this patient? 1 Facial pain, body aches, and a SPO2 of 94% 2 Length of time the symptoms have been present 3 Boggy nasal mucosal with facial tenderness 4 Low-grade fever 5 Headache and body aches

2 Length of time the symptoms have been present The condition described in this clinical scenario is highly indicative of acute bacterial rhinosinusitis (acute sinusitis). Acute sinusitis clinically is described as including symptoms such as green/yellow purulent appearing discharge, nasal obstruction, congestion, facial pain, or pressure over the affected sinus, and may also include cough, malaise, fever, or even headache. Acute sinusitis has an acute onset of symptoms, ranging from 1-4 weeks in length of duration by the time the patient presents clinically. More commonly, the origin of sinusitis is viral, but symptoms relating to this will resolve as time passes, not intensify or worsen. Bacterial sinusitis can be distinguished from viral sinusitis by symptoms persisting or worsening beyond 10 days duration. Otherwise, the symptoms described in this case and with the other answer choices can be present in both viral and bacterial sinusitis.

A 58-year-old woman presents with a 2-hour history of acute-onset severe left-sided pleuritic chest pain. Her past medical history is significant for hypertension, hyperlipidemia, and breast cancer. The pain is associated with feelings of anxiety, hemoptysis, shortness of breath, and nausea. She "feels warm," but she denies chills, palpitations, wheezing, edema, vomiting, abdominal pain, abnormal bowel habits, and dietary intolerances. She admits to a 30 pack-year smoking history, but she denies drug or alcohol use. Upon physical exam, she is found to be febrile, hypotensive, tachypneic, diaphoretic, and in acute painful distress. There is perioral cyanosis and a pleural friction rub to the left lung fields; the remainder of the exam is normal. Question What is an expected diagnostic test result for this patient? 1 Reduced plasma D-dimer levels 2 Respiratory alkalosis on arterial blood gas analysis 3 Sinus bradycardia on the electrocardiogram 4 Ventilation-perfusion matching pattern upon V/Q scanning 5 Kerley B-lines, engorged hila, and cardiomegaly on the chest X-ray

2 Respiratory alkalosis on arterial blood gas analysis This patient's presentation indicates a pulmonary embolism. The EKG is abnormal in 70% of patients with PE; the most common abnormality is sinus tachycardia. This patient exhibits tachycardia upon her physical exam. Arterial blood gases usually reveal acute respiratory alkalosis due to hyperventilation. Plasma levels of D-dimer, a degradation product of cross-linked fibrin, are elevated in the presence of a thrombus; a D-dimer <500 ng/mL using rapid ELISA provides strong evidence against venous thromboembolism. A high-probability V/Q scan for PE is most often defined as having two or more segmental perfusion defects in the presence of normal ventilation, and it is sufficient to make the diagnosis of PE in most instances. The most frequent abnormal findings of PE on the chest X-ray are atelectasis, parenchymal infiltrates, and pleural effusions, but profound hypoxia can be present with a normal chest radiograph. Kerley B-lines, engorged hila, and cardiomegaly are consistent with congestive heart failure.

A 44-year-old premenopausal Caucasian woman with a BMI of 36 presents with persistent upper right quadrant abdominal pain that radiates to the back. It has gotten so bad that she has difficulty eating any food and needs to force herself to eat. She has nausea with some episodes of vomiting. She denies bulimia but admits to anorexia. On exam, the patient has a positive Murphy's sign and tenderness to palpation in the epigastric and upper right quadrant area. Patient has a slightly elevated temperature. The physician assistant is awaiting labs and imaging. Question What is the most likely diagnosis? Answer Choices 1Acute cholecystitis 2 Chronic cholecystitis 3 Appendicitis 4 Cholangitis 5 Gastritis

Answer: 1 Acute cholecystitis Patients with acute cholecystitis present with abdominal pain accompanied by nausea and vomiting. Usually the patient consumes a large fatty meal 1 hour before the onset of symptoms, but the pain is persistent. This usually occurs due to cholelithiasis blocking the cystic duct. Patients commonly affected by cholelithiasis are those who meet the four Fs: fat, female, forty, and fertile (which describes the patient above). Chronic cholecystitis is usually an intermittent type of pain and a mild inflammation of the gallbladder. This can turn into an acute cholecystitis. Appendicitis is usually right lower quadrant pain that presents as a diffuse abdominal pain that localizes to the right lower quadrant. The patient would not have a positive Murphy's sign but would have McBurney's point, obturator sign, and psoas sign. The patient would have a fever as well. Cholangitis is defined as jaundice, fever, and right upper quadrant abdominal pain, known as Charcot's triad. Gastritis is inflammation of the stomach, which is localized in the epigastric area. This can occur for numerous reasons, but it is most commonly secondary to infectious or autoimmune causes.

A 28-year-old African American woman presents with a 4-day history of acute abdominal pain. She tells you that the pain has been achy, sore, and sometimes has a "stabbing" nature; the pain is primarily located in her lower left abdomen/pelvis. She rates the pain at a 5/10 on a pain scale. Heat and ibuprofen have been helpful in decreasing the pain. The pain is worse with intercourse and physical activity but not affected by meals. The patient reports somewhat similar pain when she had an ovarian cyst in the past. She denies fever and chills. She also denies dysuria, urinary frequency, hematuria, nausea, vomiting, bowel changes, and breast tenderness. Last menstrual period (LMP) was 4.5 weeks ago. The patient uses condoms for contraception. She has been married and monogamous for 8 years, with no prior partners. She is nulliparous. Menses have typically been regular and without severe dysmenorrhea. Her past medical history is unremarkable, with no chronic diseases and no medications. A clean-catch urine has been collected, and vitals were taken. Results are shown in the table. Weight 118 lb pH 7 Height 64" Protein Negative Pulse 90 Glucose Negative BP 116/72 mm Hg Ketones Negative Temp 97.8 °F Bilirubin Negative Urine hCG Neg. Blood (RBCs) Negative Urinalysis - dipstick Nitrite Negative Specific gravity 1.01 Leukocytes Trace Question What physical exam finding is most consistent with the suspected diagnosis? Answer Choices 1 Adnexal fullness and tenderness 2 Cervical motion tenderness 3 Chadwick's sign 4 Frothy vaginal discharge 5 Hegar's sign

Answer: 1 Adnexal fullness and tenderness This patient most likely has a follicular ovarian cyst, so the expected physical exam finding is adnexal fullness and tenderness, which would most likely be seen on the patient's left side. Follicular ovarian cysts are extremely common in women of reproductive age. An ovarian cyst will often present with acute abdominal or pelvic pain, sometimes with dyspareunia. If the cyst ruptures, peritoneal signs may be present. On bimanual exam, tenderness is classically present with ovarian cysts. Ovarian cysts vary in size, typically ranging from 2-5+ cm; they may or may not be appreciated on physical exam, depending on the patient's body habitus. This patient is thin, and an experienced clinician should be able to palpate a slightly enlarged left ovary as well as elicit tenderness on palpation. Cervical motion tenderness is the classic exam finding associated with pelvic inflammatory disease (PID). Patients with PID may present acutely or chronically with pelvic pain, fever, and a history of risk for sexually transmitted diseases. Patients with PID may also present with nausea, vomiting, change in vaginal discharge, dyspareunia, and dysuria. This patient is monogamous, uses condoms, and is afebrile, so PID is an unlikely diagnosis. Chadwick's sign is seen in pregnant patients and is a physical exam finding in which the vaginal mucosa and cervix are seen as blue in color; the color is due to hyperemia and pelvic congestion from increased blood flow. It is not associated with pelvic pain, and it would not be expected in a patient with a negative hCG. Frothy vaginal discharge is a characteristic finding of Trichomonas vaginalis infection. If the patient had this infection, her presentation may include dysuria, vaginal pruritus or odor, and abnormal vaginal discharge. Trichomonas infection is sexually transmitted and typically does not cause pelvic or abdominal pain. Hegar's sign is softening of the lower uterine segment, a physical exam finding of the lower uterus on bimanual exam; it is found in pregnant patients approximately 6-8 weeks after the LMP. This patient has a negative hCG, so she is not pregnant and she would not exhibit Hegar's sign.

A 47-year-old man presents to an urgent care center with 18 hours of abdominal pain, nausea, vomiting, and chills. He is a single construction worker, denies smoking, and has at least a 10-year history of drinking 2-4 alcoholic beverages daily. A series of lab work is performed on the patient to evaluate his abdominal pain prior to abdominal imaging. Question What laboratory results would be most indicative of the patient suffering from acute pancreatitis? Answer Choices 1 Amylase 310 U/L and lipase 760 U/L 2 Amylase 50 U/L and lipase 10 U/L 3 Aspartate aminotransferase 32 U/L and alanine aminotransferase 29 U/L 4 Aspartate aminotransferase 120 U/L and alanine aminotransferase 40 U/L 5 White blood cell count 14,000/mm3 and total bilirubin 1.8 mg/dL

Answer: 1 Amylase 310 U/L and lipase 760 U/L The correct answer is a serum amylase of 310 U/L and a serum lipase of 760 U/L, as these levels are significantly elevated, which is indicative of acute pancreatitis. The normal range for serum amylase is 30-220 U/L, and the normal range for serum lipase is 0-160 U/L. Both levels being significantly elevated is typically seen in a patient with acute pancreatitis. Serum lipase is both more sensitive and more specific than serum amylase for diagnosis of acute pancreatitis, but more so when the serum lipase is at least 3 times the normal level (as is with this case). Various other biliary and intestinal diseases can also alter these lab results. Blood work results with a serum amylase of 50 U/L and a serum lipase of 10 U/L do not indicate acute pancreatitis, as these are both normal values. Lab results of serum aspartate aminotransferase (AST) 32 U/L and serum alanine aminotransferase (ALT) 29 U/L are actually normal results for these particular lab tests. The AST and ALT can be used to identify hepatocellular diseases of the liver, but mildly increased levels can also be seen in patients with acute pancreatitis. Lab results showing serum aspartate aminotransferase 120 U/L and serum alanine aminotransferase 40 U/L can be seen in patients with various hepatocellular levels, as both are increased from normal. These results are possible in a patient with acute pancreatitis but are not the most indicative of the answer choices. Cirrhosis is a disease of the liver that can often occur in patients with large alcohol intake and often causes lab results such as these: specifically, results that show an AST level that is 3 times that of the ALT level. Lab results showing serum white blood cell count 14,000/mm3 and serum total bilirubin 1.8 mg/dL could be seen with various infections separately, but the combination of both results limits the possible disorders. The white blood cell count could be elevated in a patient with acute pancreatitis, and the serum bilirubin could also be elevated in patients with acute pancreatitis when it is associated with alcoholic hepatitis, but these results are not the most indicative of acute pancreatitis when compared to the other options.

A 60-year-old man with a past medical history of COPD, hypertension, peptic ulcer disease, and hyperlipidemia is being evaluated for a 1-hour history of severe pain in the mid-abdominal region. His history includes abdominal surgery 6 months ago for a small intestinal obstruction, and his pain emanates from his incisional site. He also states that he has shaking chills, nausea, and vomiting, but denies chest pain, shortness of breath, wheezing, or abnormal bowel habits. Upon physical exam, he is found to be hypotensive, tachycardic, diaphoretic, and in acute painful distress. The abdomen is obese, bowel sounds are hypoactive, and rebound tenderness is noted. There is an exquisitely tender 2.0 cm ventral hernia located inferior to the umbilicus that is indurated, tender to palpation, and is not reducible. Auscultation of the mass reveals the presence of bowel sounds. Question What is the most appropriate initial therapeutic intervention for this patient at this time? Answer Choices 1 Broad-spectrum antibiotics 2 Maintain a solid diet 3 Apply hot packs to the hernia 4 Oral ibuprofen for pain 5 Attempt hernia reduction

Answer: 1 Broad-spectrum antibiotics This patient's presentation is significant of a strangulated hernia. Manifestations of hernia strangulation, a medical emergency, include an exquisitely tender mass associated with systemic signs and symptoms, such as intestinal obstruction, toxic appearance, peritonitis, or meeting sepsis criteria. General surgery should be consulted immediately. Additional interventions include administering broad-spectrum IV antibiotics such as cefoxitin, providing fluid resuscitation and adequate narcotic analgesia, and obtaining preoperative laboratory studies. If there is any concern for strangulation, do not attempt hernia reduction. The reintroduction of ischemic, necrotic bowel back into the peritoneal cavity can result in subsequent perforation and sepsis. Bedside ED US, using a linear high-frequency probe with color or power Doppler of the hernia sac can be useful in these borderline cases to establish the presence or absence of blood flow.

A 42-year-old Hispanic man presents because his employee health fair lab results returned with several "out of normal" range results. He is an established patient in your practice; you have seen him 4 times for illness or minor injury over the past 10 years. He reports that he is in generally good health and feels well; he does not see any other healthcare providers. A summary of his past medical history includes: Medications: occasional over-the-counter ibuprofen for joint pain Allergies: none Surgical history: open reduction of left ankle at the age of 22 years Medical history: mild osteoarthritis Social history: patient denies the use of tobacco or illicit drugs. He drinks 3-4 beers per week. He is married and has 5 kids; he works in industrial hygiene at a lab facility. He plays rugby on the weekends as a hobby. Family history: no chronic diseases are known to the patient. His vital signs at check-in were all within normal ranges. This patient's laboratory results from the health fair are shown in the chart. Test Result Normal range Units Complete blood count (CBC) WBC 6.1 3.6-9.0 K/μL RBC 4.78 4.18-5.22 M/μL Hemoglobin 15.4 12.9-15.5 g/dL Hematocrit 45.2 34.6-50.1 % MCV 94.6 80.0-100.0 fL MCH 32.2 27.0-34.0 pg MCHC 34.1 30.0-37.0 g/dL RDW 11.7 11.0-17.0 % Platelets 462 140-440 K/μL MPV 9.9 6.5-12.0 fL WBC differential Normal Complete Metabolic Panel (CMP) Sodium 132 134-144 mmol/L Potassium 3.3 3.4-4.9 mmol/L Chloride 100 100-109 mmol/L HCO3 26 20-31 mmol/L Glucose 94 70-99 mg/dL Bun 20 18-Jul mg/dL Creatinine 1.1 0.6-1.2 mg/dL Calcium 9.9 8.8-10.5 mg/dL Albumin 3.5 3.5-5.0 g/dL Total Protein 6.3 6.4-8.2 g/dL AST (SGOT) 21 15-37 U/L ALT (SGPT) 17 May-43 U/L Alk Phosphatase 55 50-136 U/L Total Bilirubin 0.8 0.1-1.2 mg/dL Anion Gap 6 16-Jun mmol/L eGFR >60 >60 mL/min/1.73m2 Lipid Panel Cholesterol 226 ≤200 mg/dL Triglyceride 864 ≤150 mg/dL HDL 42 40-59 mg/dL LDL Unable to calculate ≤100 mg/dL VLDL 37 Jun-35 mg/dL Thyroid TSH 1.96 0.500-4.700 uIU/mL Question Based on the patient's lab results, what aspect of his history should he urgently modify? 1 Eliminate alcohol use. 2 Eliminate ibuprofen use. 3 Take a daily multivitamin. 4 Schedule yearly check-ups at the provider's office. 5 Stop all contact sports.

Answer: 1 Eliminate alcohol use. This patient has significant hypertriglyceridemia and is at risk for pancreatitis and cardiovascular disease. He should be educated to eliminate alcohol use entirely and be treated with medications to lower his triglycerides. Ibuprofen use should be avoided in patients with renal disease and ulcer disease; it should also be avoided in those with a history of gastrointestinal bleeding as well as several other conditions. This patient has a very mildly elevated blood urea nitrogen (BUN), which is an indicator of renal function. However, his creatinine and glomerular filtration rate (GFR) are normal, and his history is negative for risk factors. There is little-to-no evidence that daily multivitamins should be used in the general population. There are no indicators in this patient's history or laboratory results that he is suffering from malnutrition or specific deficiencies; therefore, a daily multivitamin is not recommended. A yearly check-up is another aspect of medical care in which there is little evidence of improved outcomes. This patient needs aggressive lowering and monitoring of his triglycerides, and a return visit in 1 year is inadequate medical care. This patient gives a history of playing rugby. If he were at high risk for a bleeding disorder (e.g., due to severe thrombocytopenia), cessation of contact sports would be wise, but this patient has slightly elevated platelets. In fact, exercise is recommended for adjunct treatment of his elevated triglycerides.

A 23-year-old man with no known significant past medical history is brought in by emergency medical services in an unconscious state. He was reported by friends to be out partying in a club and was noted to be carrying a prescription bottle during this time. The patient's father has a known history of severe spinal stenosis, for which he takes prescription opioid analgesics. His physical exam reveals slow and shallow respirations, bradycardia, hypotension, cyanosis, and miosis of both pupils. He is comatose, has diminished bowel sounds and distension with dullness to percussion over the suprapubic abdominal area, and has flaccid musculature. Question What is the preferred intervention for this patient at this time? Answer Choices 1 Intravenous naloxone 2 Intravenous nitroprusside 3 Intravenous diazepam 4 Intravenous N-acetylcysteine 5 Intravenous flumazenil

Answer: 1 Intravenous naloxone This patient presents with clinical symptomatology consistent with opioid intoxication. An initial dose of naloxone, 2 mg IV, should be administered to patients presenting with apnea or near-apnea and cyanosis. Repeated doses of 2 mg IV every 3 minutes are recommended until a maximum of 10 mg IV is reached or respiratory depression is reversed. Exposures to synthetic opioids, such as fentanyl, pentazocine, or dextromethorphan, and to sustained-release preparations may require these larger-than-ordinary doses. Other general manifestations include miosis, central nervous and cardiovascular system depression, ileus, and sphincter contraction. Intravenous nitroprusside or phentolamine would be most appropriate to treat hypertension associated with an amphetamine overdose. Benzodiazepines should not be administered since they potentiate inhibitory gamma-aminobutyric acid (GABA) neuronal activity in the CNS. Pharmacologic effects include reduction of anxiety, suppression of seizure activity, CNS depression (including possible respiratory arrest when benzodiazepines are given rapidly intravenously), and inhibition of spinal afferent pathways to produce skeletal muscle relaxation. N-acetylcysteine is indicated in a suspected acetaminophen overdose. Flumazenil is a specific benzodiazepine receptor antagonist. Benzodiazepine toxicity is suggested by the presence of lethargy, slurred speech, ataxia, coma, hypothermia, and respiratory arrest. Generally, patients with benzodiazepine-induced coma have hyporeflexia and midposition or small pupils. Coma and small pupils due to benzodiazepine overdose will not respond to naloxone but will reverse with the administration of flumazenil.

A 40-year-old Caucasian woman presents complaining of menometrorrhagia for the past 6 months. She notes associated pelvic pressure, dysmenorrhea, urinary frequency, and generalized fatigue. She is nulliparous and denies weight loss, fever, chills, chest pain, shortness of breath, abdominal pain, early satiety, nausea, vomiting, diarrhea, changes to her urine color or odor, flank pain, hematuria, or dysuria. Her physical exam is remarkable for an enlarged, mobile, 17 week-size uterus with irregular contour upon bimanual palpation. No adnexal masses, cervical motion tenderness, or vaginal discharge are noted. Urinary pregnancy test is negative. Question What medication would be most beneficial in the initial management of this patient? Answer Choices 1 Leuprorelin (Lupron) 2 Raloxifene (Evista) 3 Clomiphene (Clomid) 4 Danocrine (Danazol) 5 Estradiol (Estraderm)

Answer: 1 Leuprorelin (Lupron) The correct response is leuprorelin (Lupron). This patient's history and physical exam are most consistent with a diagnosis of uterine leiomyoma (fibroid). Pharmacologic inhibition of the growth of fibroids can be achieved by suppression of the hypothalamic-pituitary-ovarian axis through the use of gonadotropin-releasing hormone agonists (GnRH analogs) that decrease estrogen production. This reduces fibroid size by as much as 40-60%. It is also commonly used prior to a planned hysterectomy to reduce blood loss. Raloxifene (Evista) may help reduce fibroid growth, but it is unclear whether efficacy in reducing symptoms is comparable to that of other drugs. Clomiphene (Clomid) is indicated in the treatment of infertility, as it stimulates ovulation. Danocrine (Danazol) is an androgenic agonist and can suppress fibroid growth, but it is not as successful as GnRH analogs and is associated with a high rate of adverse androgenic effects (e.g., weight gain, acne, hirsutism, edema, hair loss, deepening of the voice, flushing, sweating, and vaginal dryness). Because of these side effects, this is often less acceptable to patients. Estrogen replacement agents are not suitable as treatments for uterine fibroids since uterine leiomyomas are hormonally responsive benign tumors of the uterus; they proliferate in response to estrogen.

A 48-year-old Caucasian woman presents due to feeling like she is losing her mind. She wants some tests done. Upon further questioning, she reports she is having multiple episodes daily in which she suddenly becomes very hot, flushed, and diaphoretic. These episodes last about 1 minute before resolving. She has not measured a fever. The patient reports that these episodes occur during the day and at night, during which she awakes drenched with sweat. As a result, her sleep has been poor, and she feels fatigued and irritable at both work and home. She has noticed these symptoms for about the last 2 months, and they seem to be increasing in severity. The patient has not had a period for 3 months; she recently did a home pregnancy test, which was negative. Prior to that, she had regular menses. This patient denies weight changes, palpitations, cold intolerance, bowel changes, as well as changes in her nails, skin, and hair. Although she admits irritability, she denies anxiety, depressed mood, and suicidal ideation. Her family history is remarkable for diabetes in her maternal grandfather and hypertension in her father. She is a G4P3Ab1. She denies any major psychosocial stressors recently. She drinks alcohol rarely, and she denies use of other drugs. Vitals and a urine specimen for hCG were obtained prior to the physical exam. Weight 168 lb Height 65" Pulse 72 Blood pressure 120/82 mm Hg Temperature 98.2°F Urine hCG Negative Complete screening physical exam is normal, with normal sexual development and absence of hirsutism and acne. Question Based on this patient's history and physical, what is the most reasonable management for this patient? Answer Choices 1 Offer blood tests and provide education/prescription for hormone therapy. 2 Offer blood tests and provide education/prescription for raloxifene (Evista). 3 Order cancer antigen 125 (CA-125). 4 Order pelvic ultrasound. 5 Order thyroid ultrasound.

Answer: 1 Offer blood tests and provide education/prescription for hormone therapy. Based on the history and physical, her diagnosis is perimenopause, with a classic history of vasomotor symptoms (hot flashes/flushes and night sweats). Perimenopause is defined as the time prior to menopause in which the woman's ovarian hormone output begins to decline, often beginning in the mid-40s. Menopause is defined at the point in time in which the patient has been amenorrheic for 12 months. Postmenopause occurs after that. The average age for menopause is just over 51. With a negative pregnancy test and normal physical exam at age 48 with this history, the diagnosis of perimenopause can actually be made without diagnostic studies. If tests were done, an elevated follicle-stimulating hormone is the best indicator of declining ovarian function. The most effective treatment for vasomotor symptoms is hormone therapy, and the patient's history gives no contraindications for its use. Because this patient presents to the clinic desiring tests, the clinician should offer blood tests and provide education/a prescription for hormone therapy. Raloxifene (Evista) is a selective estrogen receptor modulator (SERM), which is used in preventing and treating osteoporosis and prevention of breast cancer. It is not useful for vasomotor symptoms, and increased hot flashes are a common side effect of this medication. This would not be an appropriate medication choice for this patient. As mentioned above, blood tests could be offered but are not necessary. The cancer antigen 125 (CA-125) is a marker primarily associated with ovarian cancer. It is most useful in monitoring patients with a history of ovarian cancer for recurrence and in the evaluation of suspicious solid ovarian masses. This patient's presentation does not support a solid reason for ordering a CA-125. A pelvic ultrasound is useful in evaluating many gynecologic anatomic abnormalities, but it will not demonstrate function. In this patient's case, she presents with declining ovarian function due to perimenopause, and this is undetectable on ultrasound. Her physical exam was normal. If she had presented with pelvic complaints, an abnormal pelvic exam, or irregular or heavy menstrual bleeding, an ultrasound could have been useful. Although thyroid disorders can cause menstrual cycle changes and fatigue, ordering a thyroid ultrasound is not indicated in this case. A more appropriate way to rule in or rule out thyroid disease is with appropriate history, physical exam, and a serum thyroid-stimulating hormone level. This patient denies other symptoms of hypothyroidism (weight gain, dermatologic changes, cold intolerance, constipation, and depressed mood). Hypothyroidism is not associated with hot flashes and night sweats.

A 32-year-old African American woman with no significant past medical history has been referred to a pulmonologist; she presents with a 2-month history of progressive dyspnea. She notes associated low-grade fever, malaise, joint pain, and swollen neck glands. She denies a history of travel, cigarette smoking, drug use, or sexually transmitted diseases (she has not been sexually active in the past year). All other reviews of systems are negative. Her physical exam reveals tender nodular formations on her anterior lower extremities, parotid enlargement, hepatosplenomegaly, and cervical lymphadenopathy. Her vital signs, heart, and lungs are unremarkable. Diagnostic testing reveals leukopenia, increased ESR, hypercalcemia, hypercalciuria, elevations of serum ACE levels, and bilateral hilar adenopathy with diffuse reticular infiltrates. ANCA, ANA, and rheumatoid factor tests are negative. Histological assessment confirms the presence of noncaseating granulomas. Question What is the most likely diagnosis? Answer Choices 1 Sarcoidosis 2 Tuberculosis 3 Wegener's granulomatosis 4 Pneumocystis jirovecii pneumonia 5 Idiopathic pulmonary fibrosis

Answer: 1 Sarcoidosis This patient's presentation is most consistent with sarcoidosis. It is a systemic disease of unknown etiology most commonly affecting African American female patients. Onset is usually in the 20s and 30s, with patients presenting with constitutional symptoms such as fever and malaise. Other manifestations include insidious dyspnea, erythema nodosum, parotid gland enlargement, hepatosplenomegaly, neuropathy, arthritis, and lymphadenopathy. Labwork demonstrates leukopenia, increased ESR, hypercalcemia, hypercalciuria, and increased angiotensin-converting enzyme levels. Chest X-ray findings commonly show bilateral hilar adenopathy, but they may also demonstrate diffuse reticular infiltrates, acinar shadows, nodules, and cavitations and effusions, rarely. Biopsy findings reveal noncaseating granuloma formations. Tuberculosis is a caseating granulomatous infection characterized by fever, followed by night sweats, malaise, fatigue, and weight loss. Productive cough, hemoptysis, dyspnea, and pleuritic chest pain develop as the infection spreads within the lungs. Patients at greatest risk include the homeless, immigrants, those with HIV, those who abuse alcohol and/or drugs, and elderly patients. Radiographic imaging typically reveals cavitary or noncavitary lesions in the upper lobe or superior segment of the lower lobe of the lungs. Wegener's granulomatosis is characterized by granulomatous inflammation, vasculitis, and necrosis. Patients present with persistent rhinorrhea, unusually severe nasal obstruction, epistaxis, bloody or brown nasal crusting, hearing loss, facial nerve palsy, and inner ear involvement, vestibular dysfunction (nausea, vertigo, and tinnitus). In the mouth, gum inflammation ("strawberry gums") and tongue ulcers are common. Approximately 80% of patients have pulmonary lesions. Radiographic findings most commonly include pulmonary infiltrates and nodules. Symptoms consist of cough, hemoptysis, dyspnea, and pleuritic chest pain. Renal disease is the most ominous clinical manifestation and is of a rapidly progressive glomerulonephritis, typified by hematuria, red blood cell casts, proteinuria (usually non-nephrotic), and rising serum creatinine. Labwork reveals positive antineutrophil cytoplasmic antibodies (ANCAs, especially c-ANCA), rheumatoid factor, and elevations in the ESR. Histologically, vascular necrosis begins as clusters of neutrophils within the blood vessel wall (microabscesses) that degenerate and become surrounded by palisading histiocytes. Pneumocystis jirovecii (formerly carinii) pneumonia (PCP) is the most common opportunistic infection in HIV-infected patients. Clinical suspicion for PCP pneumonia in any HIV patient presenting with reports of dyspnea and nonproductive cough should remain high. Presentations can be indolent, acute, or subacute; associated symptoms include fever, fatigue, anorexia, weight loss, and chest pain. The CBC is usually normal except for lymphopenia, while serum LDH is often elevated. Arterial blood gases most often reveal a respiratory alkalosis, PO2 ≤70 mm Hg, and an increased A-a gradient ≥35 mm Hg. Radiographic chest findings are variable, with diffuse interstitial alveolar infiltrates being common. Idiopathic pulmonary fibrosis typically presents in the 40s-60s, with a slight male predominance. There is no known causative agent. Idiopathic pulmonary fibrosis (interstitial pulmonary fibrosis or cryptogenic fibrosing alveolitis) is an uncommon disease of unknown etiology. It is marked by chronic inflammation of alveolar walls, resulting in diffuse and progressive fibrosis and destruction of normal lung architecture. This process produces a restrictive defect with altered ventilation and increased work of breathing and destructive and obliterative vascular injury that can severely impair normal pulmonary perfusion and gas exchange. It is cytokine-mediated via a proliferation of lymphocytes and neutrophils; fibroblasts also release cytokines while producing matrix molecules, including collagen. The usual presentation of idiopathic pulmonary fibrosis is with the insidious onset of progressive dyspnea, generally accompanied by a dry and persistent hacking cough. Fever and chest pain are generally absent. With disease progression, dyspnea often worsens, even occurring at rest. Digital cyanosis and clubbing are commonly seen. In the later stages of the disease, increasing pulmonary hypertension can lead to right heart failure and peripheral edema. The characteristic radiographic findings include small lung volumes, with increased densities more prominent in the lung periphery. Fibrosis surrounding expanded small airspaces is seen as honeycombing.

A 22-year-old nulligravida presents to your office with a 3-day history of green-yellow frothy malodorous discharge; her last menstrual period was 2 weeks ago. She has also experienced mild pelvic pain during the same period of time. She is sexually active with two male partners and she uses condoms occasionally. On wet mount, motile protozoans and white blood cells are noted. Question What may be seen with this type of vaginitis? Answer Choices 1 Strawberry cervix 2 Pseudohyphae 3 Ferning 4 Positive whiff test 5 Clue cells on wet mount

Answer: 1 Strawberry cervix The patient has trichomoniasis, which is caused by the protozoan Trichomonas vaginalis. Vaginal discharge is often profuse and may be accompanied by mild pelvic pain and/or dyspareunia. In 10% of cases, a strawberry cervix may be seen. Pseudohyphae may be seen in wet mount examinations of vulvovaginal candidiasis. Ferning may be seen in microscopic examination of vaginal secretions in a pregnant patient with ruptured membranes. A positive whiff test is noted when vaginal secretions from a patient with bacterial vaginosis are mixed with potassium hydroxide (KOH), liberating amines, which have a characteristic fishy odor. The whiff test is 1 of the 4 diagnostic criteria for the diagnosis of bacterial vaginosis.

A 75-year-old African American man presents with a 5-month history of gradually progressive dyspnea that is especially pronounced when climbing stairs. He also has been noticing that his ankles and lower legs have "gotten larger" over roughly the same time period, which no longer allows him to fit into his sneakers. He denies fever, chills, chest pain, palpitations, cough, pleurisy, calf pain, abdominal complaints, sick contacts, or travel. His psychosocial history is noteworthy for chronic alcohol use. His physical exam reveals bibasilar rales, JVD of 5 cm, an S3 gallop, a holosystolic murmur at the apex that radiates to the left axilla, and 2+ pitting edema to the level of the mid-calves bilaterally. A bedside echocardiogram was remarkable for biventricular enlargement. Question What additional physical exam finding would be expected in this patient? Answer Choices 1 Tachycardia 2 Fever 3 Asymmetric upper extremity blood pressures 4 Warm, moist skin 5 Acanthosis nigricans

Answer: 1 Tachycardia This patient's presentation is significant for dilated cardiomyopathy. Dilated cardiomyopathy occurs more often in the African American population than in the Caucasian population, and it occurs in men more frequently than women. Often, no cause can be identified, but chronic alcohol abuse, major catecholamine discharge, myocarditis, postpartum state, and doxorubicin are frequent causes. Chronic tachycardia may also precipitate a dilated cardiomyopathy that may improve over time if rate control can be achieved. Amyloidosis, sarcoidosis, hemochromatosis, and diabetes may rarely present as dilated cardiomyopathies, as well as the more classic restrictive picture. Signs and symptoms of heart failure develop gradually and include rales, an elevated jugular venous pressure (JVP), cardiomegaly, S3 gallop, murmurs of functional mitral or tricuspid regurgitation, peripheral edema, or ascites. In severe CHF, Cheyne-Stokes breathing, pulsus alternans, pallor, and cyanosis may be present. Sinus tachycardia is common. Other common ECG abnormalities include left bundle branch block and ventricular or atrial arrhythmias. The chest radiograph reveals cardiomegaly, evidence for left and/or right heart failure, and pleural effusions. Fever is a sign of infection which is not identified in this patient. Asymmetric upper extremity blood pressures is a classic presentation of coarctation of the aorta. The patient's clinical findings do not support this diagnosis. Warm, moist skin would be rare since the patient denied fever and chills. Acanthosis nigrans is associated with insulin resistance, which is not the presentation of this patient.

A 3-month-old male infant presents for a routine evaluation. His mother states that the child is gaining weight, is feeding appropriately, and has been without fever, chills, dyspnea or other abnormal objective signs. Upon physical examination, the examiner noticed a loud, harsh holosystolic murmur in the left third and fourth interspaces along the sternum that was associated with a systolic thrill. There were no other abnormalities. Question What is correct regarding this patient's diagnosis? Answer Choices 1 The smaller the defect is, the louder the associated murmur. 2 This is a common finding in the adult population. 3 A right-to-left shunt is expected in this patient. 4 This patient is expected to have a lower life expectancy. 5 Small shunts require immediate surgical repair.

Answer: 1 The smaller the defect is, the louder the associated murmur. This patient's presentation represents a ventricular septal defect (VSD). Congenital VSDs occur in various parts of the ventricular septum. The smaller the defect, the greater the gradient from the left ventricle to the right ventricle and the louder the murmur. Newly arising VSDs are uncommon in adults. A left-to-right shunt is present unless there is associated right ventricular hypertension. Patients with the typical murmur as the only abnormality have a normal life expectancy, but they have an increased risk of associated bacterial endocarditis in smaller shunts. Membranous and muscular septal defects may spontaneously close in childhood as the septum grows and hypertrophies; small shunts in asymptomatic patients do not require surgery or other intervention.

A 32-year-old man with a past medical history of allergic rhinitis and asthma that is well-controlled complains of recurrent pruritus associated with an erythematous rash in the flexural areas of his elbows and knees. The lesions seem to become worse when he scratches them and when he is under stress. He denies any recent insect bites, travel, fever, chills, new clothing, or detergent use. Physical examination reveals rough-appearing erythematous plaques in the bilateral antecubital and popliteal fossae, with areas of excoriations within the lesions. Question What intervention is most appropriate at this time? Answer Choices 1 Triamcinolone 0.1% applied to the lesions once or twice daily 2 Cephalexin 500 mg by mouth every 12 hours 3 Acyclovir 200 mg by mouth 5 times per day 4 Tacrolimus ointment 0.03% applied to the lesions twice daily 5 Prednisone 40 mg by mouth daily tapered over 2-4 weeks

Answer: 1 Triamcinolone 0.1% applied to the lesions once or twice daily The dermatologic lesions described in this patient, along with his coexisting past medical history, are most indicative of an eczematous eruption. Corticosteroids should be applied sparingly to the dermatitis once or twice daily and rubbed in well. In general, one should begin with triamcinolone 0.1% or a stronger corticosteroid and then taper to hydrocortisone or another slightly stronger mild corticosteroid. The presence of fissures, crusts, erosions, or pustules may indicate staphylococcal infection. Antistaphylococcal antibiotics should be given systemically if an infection is suspected. Eczema herpeticum, a generalized herpes simplex infection manifested by monomorphic vesicles, crusts, or scalloped erosions superimposed on atopic dermatitis or other extensive eczematous processes, is treated successfully with oral acyclovir. Immunosuppressant therapies, such as tacrolimus, can be effective in managing atopic dermatitis when applied twice daily. The agents should be used sparingly and only in locations where less expensive corticosteroids cannot be used. Tacrolimus and pimecrolimus both have black-box warnings for patients at high risk for lymphoma (those with HIV, iatrogenic immunosuppression, prior lymphoma). Systemic corticosteroids, such as prednisone, are indicated only for severe acute exacerbations.

A 24-year-old man with no significant past medical history presents with a 3-month history of progressive hearing loss of his left ear. He states that he has the greatest difficulty in hearing high-pitched sounds. He denies trauma, recent travel, sick contacts, pressure changes, headache, nausea, otorrhea, otalgia, fever, chills, vertigo, swollen glands, rashes, sore throat, vision changes, or rhinitis. He states that he enjoys listening to music on his personal music device and attends concerts frequently. An otoscopic speculum exam reveals no observable abnormalities. Question What diagnostic test would be most useful in identifying the cause of this patient's hearing loss at this time? Answer Choices 1 Weber test 2 Cochlear biopsy 3 Facial X-rays 4 CT scan of the head 5 Complete blood count

Answer: 1 Weber test This patient's presentation is most consistent with sensorineural hearing loss due to exposure to loud noise. Tuning forks are useful in differentiating conductive from sensorineural hearing losses. In this patient, the Weber test is expected to reveal a sound that lateralizes to the good or unaffected ear, as cochlear nerve damage impairs transmission to the affected ear. The following, confirmatory Rinne test will demonstrate air conduction greater than bone conduction, the normal pattern. The normal pattern prevails since the cochlear nerve is less able to transmit impulses regardless of how the vibrations reach the cochlea. Collectively, the Weber and Rinne tuning fork tests and a fistula test using pneumatic speculum must be performed. Audiometry is important to perform and includes pure-tone, speech tests, and immittance (tympanometry and acoustic reflex) tests. A cochlear biopsy is invasive and not routinely performed. Facial X-rays have no role in the workup of hearing loss. According to 2012 guidelines from the American Academy of Otolaryngology-Head and Neck Surgery Foundation, computed tomography scans are not helpful and expose the patient to ionizing radiation. CT scans are therefore not recommended in the initial evaluation of patients with presumptive sensorineural hearing loss. Routine, non-targeted laboratory testing is not recommended.

A 32-year-old man with no significant past medical history presents with a 2-month history of increased dyspnea upon exertion; the dyspnea becomes apparent after walking 10 city blocks. He denies associated symptoms, such as fever, chills, changes in weight, chest pain, abdominal pain, nausea, and vomiting. He also denies any history of cigarette smoking, occupational risk factors, sick contacts, and recent travel. His physical exam reveals normal vital signs and no distension of his jugular vein, but there is a prominent right ventricular impulse along the lower-left sternal border that is associated with a palpable pulmonary artery. There is also a mid-systolic ejection murmur at the upper left sternal border that does not vary in intensity with respiration. There is a fixed split second heart sound. The remainder of his examination is normal. Question What is the most likely diagnosis? Answer Choices 1 Patent ductus arteriosus 2 Atrial septal defect 3 Ventricular septal defect 4 Mitral stenosis 5 Pulmonic stenosis

Answer: 2 Atrial Septal Defect This patient's presentation suggests an atrial septal defect. Manifestations are caused by the quantity of blood shunted from the systemic to pulmonary circulation or from the left atrium to the right atrium. The size of the shunt depends on the size of the defect and the relative compliance of the right and left ventricle. The murmur associated with ASD is due to an increase of blood flow across the pulmonic valve, which does not vary in intensity with respiration. If pulmonary hypertension is present, P2 is increased and a high-pitched murmur of pulmonary regurgitation (Graham Steell murmur) may be audible. Signs of right ventricular failure with elevated jugular venous pressure and venous congestion may be apparent in the later stages of this disease. Patent ductus arteriosus is characterized by an abnormal patency of the ductus arteriosus, which diverts blood from the right side of the heart to the systemic circulation during fetal life. Blood typically shunts from the higher-pressured left side (systemic circulation) to the lower-pressured right side (pulmonary circulation). This murmur is described as a hollow, machinery-like murmur that is continuous throughout the cardiac cycle. Ventricular septal defect is the most common congenital cardiac malformation. Blood flows from the left ventricle to the right ventricle and presents as a harsh, blowing holosystolic murmur with a thrill localized to the fourth left intercostal space. This murmur may decrease with Valsalva and handgrip. Pulmonary stenosis is a crescendo-decrescendo in quality and is characterized by a harsh systolic murmur in the second and third left interspaces; it has a medium pitch. It may radiate to the left shoulder and neck; if severe, it may be associated with a widely split S2 with a diminished or inaudible pulmonic component. The murmur of mitral stenosis is a decrescendo low-pitched rumble that is usually limited to the apex, with an accentuated opening S1 and opening snap that follows S2.

A 62-year-old woman presents with bright red gross hematuria for the past 2 months. She states it is painless but persistent. She denies other symptoms. She has no chronic medical problems. Upon further questioning, she admits to a 50 pack-year smoking history, and she states she is currently retired from her job in a rubber factory. Vital signs are within normal limits, and physical exam is normal. Urine dipstick only shows red blood cells that are too numerous to count and urine cultures are negative. Question What is the most likely diagnosis? Answer Choices 1 Acute cystitis 2 Bladder cancer 3 Renal cell carcinoma (RCC) 4 Urethritis 5 Ureteral calculi

Answer: 2 Bladder cancer Hematuria is the most common presenting sign of urinary tract cancer, and bright red gross hematuria is usually of lower urinary tract origin. Silent or painless hematuria suggests tumor or renal parenchymal disease, so the clinical picture points to cancer, with bladder cancer as the most likely diagnosis. Smoking and exposure to industrial dyes or solvents (like in a rubber plant) are risk factors for bladder cancer. Bladder cancer is the second most common urologic cancer, and the mean age at diagnosis is 65. It is more common in men than women (2.7:1), and 98% of primary bladder cancers are epithelial malignancies (majority urothelial cell carcinomas). Ordering cytology of the urine sample is often helpful with higher grade and stage bladder cancers. Patients can become anemic with chronic blood loss, so a CBC is justified. Diagnosis is made by cystoscopy with biopsy. Acute cystitis typically presents with irritating voiding symptoms (frequency, urgency, dysuria), suprapubic discomfort, and possible hematuria. Urinalysis will show pyuria, bacteriuria, and varying degrees of hematuria. Urine cultures will show specific organisms. Renal cell carcinoma can present with flank pain, hematuria, persistent back pain, and an abdominal mass; also, it can be found incidentally on CT scan. RCC is more common in men than women (2:1), and it has a peak incidence in ages 50-60. This could be a possible option for diagnosis, but the significant history of smoking and previous work history points more toward bladder cancer. Urethritis is inflammation of the urethra that presents with urethral discharge, dysuria, and itching. Urethritis is most often caused by an STD. This patient does not have a history of unprotected sexual intercourse or any other symptoms that would indicate this diagnosis. Ureteral calculi can present with hematuria, but it also typically presents with flank or abdominal pain. If the stone is in the ureter, it often causes some hydroureter with or without hydronephrosis, both of which cause some pain or discomfort. The patient has no past history of forming stones.

A 13-year-old girl is drowsy and unable to answer questions. Her mother says that she has been extremely thirsty lately and urinates frequently. Her father notes that the patient has also been fatigued. There is a fruity odor to the patient's breath. Blood gases are drawn and reveal the following: pH 7.3 CO2 32 HCO3 17 Question What is the most likely cause of the patient's condition? Answer Choices 1 Frequent vomiting 2 Diabetes mellitus 3 Myasthenia gravis 4 Ethylene glycol ingestion 5 Salicylate ingestion

Answer: 2 Diabetes mellitus The patient's blood gas results indicate metabolic acidosis. Given her presentation of polydipsia, polyuria, fatigue, and fruity odor to her breath, she is likely suffering from diabetic ketoacidosis from undiagnosed diabetes mellitus. Frequent vomiting would lead to metabolic alkalosis from hydrogen loss. Myasthenia gravis would be associated with respiratory acidosis due to the associated decrease in pulmonary function and therefore decreased clearance of CO2. Ethylene glycol ingestion is associated with metabolic acidosis, but the patient's presentation is classic for diabetic ketoacidosis. Salicylate ingestion is associated with respiratory alkalosis.

A 47-year-old woman with a history of obesity presents to the emergency department due to pain in her right ankle after a fall 2 days ago. Upon physical exam, she has pain and tenderness at the tip of the medial malleolus and has the inability to bear weight for at least 4 steps. The patient does not present with pruritus or any rashes; however, the ankle does appear swollen. Additionally, the patient's foot does appear to be neurologically intact. Furthermore, she does not present with a fever or erythema of the ankle. According to the Ottawa Ankle Rule, she qualifies for X-rays. Question What first-line treatment can be used to help treat this condition? Answer Choices 1NSAID 2 Immobilize in temporary cast/splint 3 Opioid 4 Antifungal 5 Antibiotic

Answer: 2 Immobilize in temporary cast/splint Immobilize in temporary cast/splint is the correct treatment, as this patient has a stable fracture that requires temporary immobility, rest, ice, and elevation. If the patient is in pain, NSAIDs can be taken as well. NSAID is a treatment for the patient's fracture; however, it is not the first-line treatment and therefore an incorrect answer to this problem. Opioid would be used as adjunctive therapy, not as a first line of treatment An antifungal would not help treat a patient with a fracture, and the patient does not present with any pruritus or rashes. The patient does present with an infection of the foot but does not present with a fever or erythema of the ankle; therefore, an antibiotic would not be the proper treatment for this patient.

A 2-month-old male infant is being seen for a routine examination by his pediatrician. His mother admits to not following recommendations and has not had him seen by the pediatrician since his hospital discharge. During the genitourinary examination, the pediatrician cannot palpate the testis on either side of the scrotum. The pediatrician is concerned that the infant has bilateral cryptorchidism (undescended testes) but needs to make sure testes are present somewhere above the scrotum. Question What laboratory testing and imaging study combination would the pediatrician order? Answer Choices 1 Luteinizing hormone, follicle-stimulating hormone, and testosterone levels followed by CT scan 2 Luteinizing hormone, follicle-stimulating hormone, and testosterone levels followed by ultrasonography 3 Human chorionic gonadotropin stimulation test and testosterone levels followed by ultrasonography 4 Human chorionic gonadotropin stimulation test and testosterone levels followed by X-ray 5 Human chorionic gonadotropin stimulation test and testosterone levels followed by CT scan

Answer: 2 Luteinizing hormone, follicle-stimulating hormone, and testosterone levels followed by ultrasonography Luteinizing hormone (LH), follicle-stimulating hormone (FSH), and testosterone levels followed by ultrasonography is correct. In male infants under the age of 3 months, LH, FSH, and testosterone levels are helpful in determining whether there are testes present. Ultrasonography has a sensitivity of 76%, a specificity of 100%, and an overall accuracy of 84% when diagnosing undescended testes that are nonpalpable on examination. MRI would have also been a good imaging choice, as it has a sensitivity of 86%, a specificity of 79%, and an overall accuracy of 85%. Ultrasonography is both easier and a lower risk to perform on infants and children, however, making it a more popular choice in many cases. CT scan findings in children, when used in the diagnosis of nonpalpable undescended testes, are historically not reliable. The HCG stimulation test is done by administering 2000 IU of HCG daily for 3 days and checking testosterone levels pre- and post-stimulation. This helps to determine the presence or absence of testicular tissue, but this test is reserved for infants older than 3 months. X-ray is not used at all in the diagnosis of cryptorchidism.

A 41-year-old woman presents due to worsening symptoms. She was diagnosed with idiopathic pulmonary hypertension about 2 years prior to presentation; she is on home oxygen therapy. She has longstanding fatigue and dyspnea, but she is now experiencing profound dyspnea with exertion, swelling in her ankles, some discomfort in her right upper abdomen, and the inability to breathe well when lying down. She has always been thin, but her weight has increased by 10 pounds in the last month. She denies fever and chills. She recently had an electrocardiogram (ECG), but she has not seen a healthcare provider to discuss the results. The ECG report indicates peaked p waves, right axis deviation, and tall r wave in V1. Question What is the most appropriate intervention for her current condition? Answer Choices 1 Prescribe a calcium channel blocker, such as verapamil. 2 Prescribe a diuretic, such as furosemide. 3 Prescribe a fluoroquinolone, such as levofloxacin. 4 Prescribe a lipase inhibitor, such as orlistat. 5 Prescribe a thiazolidinedione, such as pioglitazone.

Answer: 2 Prescribe a diuretic, such as furosemide. This patient is presenting with a progression of a primary pulmonary disease (pulmonary hypertension) into cor pulmonale, which is also known as pulmonary heart disease. Cor pulmonale, when moderate to severe, will present with signs and symptoms of right heart failure, such as the severely fluid overloaded state. In addition to treatment of the underlying pulmonary disorder, cor pulmonale is treated much like right-sided heart failure is treated. It would be most appropriate to prescribe a diuretic, such as furosemide, at this time. If possible, this patient should also be referred to a cardiopulmonary specialist. A calcium channel blocker, such as verapamil, can actually worsen right ventricular function, which is already diminished in this patient. The ECG findings indicate right ventricular hypertrophy. A fluoroquinolone, such as levofloxacin, would be an appropriate choice if this patient's dyspnea were due to bacterial pneumonia. She is afebrile, however, and her fluid overload symptoms and ECG findings are not typical in cases of pneumonia. A lipase inhibitor, such as orlistat, might be considered for weight loss in a patient with obesity and the inability to lose weight. This patient reports a relatively recent weight gain, however, which is attributable to fluids. A thiazolidinedione, such as pioglitazone (Actose), is contraindicated in a patient with symptomatic heart failure. In essence, this patient has right-sided heart failure due to her pulmonary hypertension. Actose is used for diabetes; there is no evidence that this patient has diabetes.

An 82-year-old man presents for evaluation of an itchy hive-like rash on his abdomen. He has had it for months. Over the last few days, the patient reports that the character of the rash has changed: it now resembles blisters. Other than some skin irritation, he feels healthy. He has tried multiple over-the-counter topical treatments, but they have been unsuccessful in treating the rash. He denies any changes in soaps, lotions, laundry detergents, or anything else that may have come in contact with his skin. He also denies unusual travel, pets, or hobbies. No close contacts have reported a similar condition. He takes no medications, and he does have any chronic illnesses. On physical exam, multiple clusters of bullae are noted across the trunk bilaterally, with some distribution on both anterior and posterior surfaces. There are still some remaining pink-red lesions; they are scattered among the bullae. The bullae are 1-3 cm in size; they are tense and do not easily rupture. They do not extend into normal skin with pressure. Pressure on the normal skin does not produce a blister. The remainder of his physical exam is normal. Question What test would confirm the most likely diagnosis? Answer Choices 1Serology for parvovirus B19 2 Skin biopsy with immunofluorescence testing 3 Skin scraping with potassium hydroxide 4 Viral culture of fluid inside bullae 5 Wood lamp examination

Answer: 2 Skin biopsy with immunofluorescence testing This patient presents with bullous pemphigoid (BP), a benign autoimmune skin disorder characterized by the presence of bullae (or blisters). BP is relatively rare and tends to affect the elderly. The preceding rash is typically pruritic, and it appears as urticarial plaques, possibly resembling erythema multiforme. This stage may last for months, then the skin blisters. When the blisters rupture (typically within 1 week), the skin heals. Unless the BP is extensive and generalized, the prognosis is good. Skin biopsy with direct and indirect immunofluorescence and light microscopy is the best test for confirming BP. Serology for parvovirus B19 is the appropriate confirmatory test for patients suspected to have fifth disease (erythema infectiosum), which presents with the characteristic "slapped cheek" rash. It is not helpful in cases of BP. A skin scraping with potassium hydroxide (KOH) would be helpful in diagnosing a fungal skin condition. While pruritus can be associated with fungal infections, this patient's presentation with bullae does not suggest a fungal etiology. A viral culture of the fluid inside the bullae would not reveal any virus, as BP is an autoimmune condition. Herpes family infections could be on the differential, especially with a blister-like appearance, but herpes vesicles are typically much smaller, and none of the herpes infections fit the presentation and distribution of this patient's "rash." A Wood lamp examination can be helpful in evaluating several dermatologic conditions, such as vitiligo, as well as some bacterial and fungal infections. It uses ultraviolet light. A Wood lamp exam can also be performed on the eyes when checking for corneal abrasions, but it is not helpful in cases of BP.

A 22-year-old Asian man is presenting to a hematology clinic for the first time to seek management for his alpha thalassemia (hemoglobin H disease type). He has a strong family history of the disorder and was previously being managed by a different hematologist. Today, he receives a thorough history and physical examination, as well as various blood tests. Question What physical examination finding is most likely to be present? 1 Hepatomegaly 2 Splenomegaly 3 Decreased vibratory sensation 4 Positive Murphy's sign 5 Retinopathy

Answer: 2 Splenomegaly The thalassemias are a hereditary form of microcytic anemia characterized by a reduction in the synthesis of globin chains. Alpha thalassemia, specifically, occurs when a gene or genes related to the alpha globin protein is changed or missing. Alpha thalassemia occurs most often in people from Southeast Asia, China, the Middle East, and Africa. Alpha thalassemia can be further differentiated depending on the number of alpha globin genes that are present. Normally, there are four copies of the alpha globin chain. Patients with three alpha globin genes are silent carriers and are hematologically normal. Patients with two alpha globin genes have the alpha thalassemia trait but are clinically normal with only a mild microcytic anemia. If only one alpha globin gene is present, the patient is said to have hemoglobin H disease characterized by chronic hemolytic anemia. Symptoms may include fatigue, shortness of breath, and jaundice. Pallor and splenomegaly are common physical examination findings. Hepatomegaly is liver enlargement that can be seen in various disorders but is not common in patients with alpha thalassemia. Decreased vibratory sensation is a neurologic examination abnormality that can be found in various types of patients with neurologic disorders, but alpha thalassemia is not typically associated with any neurologic findings. Positive Murphy's sign is associated with pain upon deep palpation during expiration during the abdominal examination of the upper right quadrant. It is typically positive when a patient has an inflamed gallbladder or cholecystitis. Cholecystitis is not an associated consideration with alpha thalassemia, but it can be seen in patients with hereditary spherocytosis. Retinopathy during a fundoscopic eye examination is not a common finding in patients with alpha thalassemia, but it can be seen in patients with sickle cell anemia. The appearance is similar to that of patients with diabetic retinopathy.

An 18-year-old man presents with a rash. He states that the rash began a few weeks ago and it has worsened over the past 2 weeks. He denies fever, chills, nausea, vomiting, or weight loss. He also denies recent travel or illness. He has no significant past medical history and is otherwise very healthy. Physical exam reveals a well-developed, well-nourished man in no acute distress. He has areas of hyperpigmentation on his back and chest. A scraping taken from the back area shows orange fluorescence under UV light. Question What is the most likely cause of this patient's symptoms? Answer Choices 1 Vitiligo 2 Tinea versicolor 3 Psoriasis 4 Tinea unguium 5 Seborrheic dermatitis

Answer: 2 Tinea versicolor Tinea versicolor is a fungal infection common in adults and adolescents. The most commonly affected areas include the chest, back, and shoulders. Occasionally, it can be found on the face. It causes the affected skin to change color, becoming either lighter or darker. It was believed to be caused by a yeast called Malassezia furfur, but recent evidence points to Malassezia globosa as the cause. Tinea versicolor can recur, so treatment may need to be repeated in the future. The affected skin becomes reddish-brown to brown or may be light in color. Initially, the lesions are well-defined round-to-oval scaly macules. Over time, they tend to coalesce and form patches with various amounts of shading. The colors can be darker or lighter than the unaffected skin. Darker patches may disappear shortly after treatment is started, but lighter patches may take a long time to go away. The skin discoloration is not permanent; the color will eventually return to normal. Diagnosis can be confirmed by using an ultraviolet light (Wood light). The affected areas usually fluoresce and appear to be orange in color. If they do not fluoresce, the skin will appear darker than normal skin. A scraping of the skin will show the presence of hyphae in a characteristic "spaghetti and meatballs" appearance when exposed to potassium hydroxide. Diagnosis can also be confirmed through microscopic analysis. A scraping of the area placed in potassium hydroxide solution will show hyphae if a fungal infection is present. Tinea versicolor can be treated with several preparations that are applied to the skin. Over-the-counter preparations usually contain miconazole, ketoconazole, or clotrimazole; they can be found in shampoo or cream form. There are also prescription strength versions of these preparations. Oral medications (e.g., itraconazole or ketoconazole) also exist. Vitiligo causes a loss of pigmentation. Psoriasis causes raised scaly patches that do not fluoresce under UV light. Tinea unguium is a fungal infection of the nails. Seborrheic dermatitis causes patches of thick, scaly, crusty skin; it occurs mainly on the scalp. It can also appear in areas with oil glands (e.g., the folds of skin as in those of the face, groin, and armpits).

A 35-year-old man presents with paralysis and confusion. His wife states he had been working in the yard about 3 days ago and suddenly began to feel unwell. She states that it has progressed to the point where he appears quite confused, is extremely agitated, and does not seem to be able to close his right eye completely or smile on the right side of his face. The patient has a rash on the right arm that does not seem to be pruritic. Physical examination of the patient reveals findings of altered mental status, a cranial nerve palsy of the facial nerve (cranial nerve VII), exaggerated deep tendon reflexes, and the presence of a maculopapular rash on the right forearm. Question What is the most likely source of this patient's condition? Answer Choices 1 Mumps 2 West Nile virus 3 St. Louis virus 4 Rabies 5 Varicella zoster virus

Answer: 2 West Nile virus This patient has encephalitis secondary to exposure to West Nile virus. Patients with encephalitis, regardless of the cause, will present with an altered mental status that has a huge variability (from subtle deficits to 100% unresponsiveness). Meningeal irritation may also be seen (e.g., photophobia and nuchal rigidity). Seizures will also be seen, as well has hemiparesis, cranial nerve palsies, exaggerated deep tendon—or even pathologic—reflexes. Patients will appear confused, agitated, or obtunded. Patients with West Nile virus encephalitis may be misdiagnosed as having Guillain-Barré syndrome because of the presence of the symptom of flaccid paralysis that is specific to this infection. Another key characteristic of this encephalitis is the presence of a non-pruritic maculopapular rash, estimated to be seen in up to 50% of patients with this type of infection. This patient most likely has encephalitis secondary to West Nile virus. A patient who is unvaccinated and presents with parotitis and altered mental status will more likely have encephalitis secondary to mumps. If a patient has tremors of the eyelids, tongue, lips, and extremities, the most likely cause is St. Louis virus. Findings such as hydrophobia, pharyngeal spasm, and hyperactivity would be seen in a patient with rabies-induced encephalitis. A patient who has encephalitis from varicella zoster virus will more commonly have the dermatomal pattern of grouped vesicles that is typically seen in a zoster infection.

A 23-year-old woman presents to the emergency department due to double vision; pain upon moving the right eye; and numbness of the cheek, nose, and right side of the eye. She was playing baseball with her team and the ball hit her in the right eye. Upon physical exam, swelling, tenderness, numbness of the nose, and epistaxis are present Question What is the most likely diagnosis? Answer Choices 1Blepharitis 2 Blowout fracture 3 Conjunctivitis 4 Corneal abrasion 5 Stye

Answer: 2 blowout fracture The clinical picture is suggestive of blowout fracture, commonly known as an orbital floor fracture. The patient presents with a history of being hit in the right eye and presents with double vision, and numbness of the cheek and right side of eye. Additionally, X-ray would confirm a fracture. Blepharitis is not the correct answer. It is when oil glands become clogged or irritated. Additionally, a patient would present with itchy eyelids, watery eyes, a feeling that something is in their eye, or crust on the eyelashes. These symptoms are not present in this patient. Conjunctivitis is not the correct answer because conjunctivitis presents with burning, itching, irritation, and discharge, all of which is not present in this patient. Corneal abrasion is not the correct answer because it does not present with swelling, double vision, and numbness of the cheek. Instead, it is diagnosed with fluorescein stain and described as very painful, due to the scratch on the surface of the cornea. Stye is not the correct answer, as it is when bacteria gets into the oil gland in the eye lids. Additionally, it creates a red bump that is located closer to the eyelashes. It can make your eye feel watery, or as though something is in your eye. All of these symptoms are not present in the case.

A 35-year-old woman presents with slowly progressive right-sided hearing loss, tinnitus, and continuous vertigo. Patient's Weber test reveals lateralization to the left ear. Question What is the most likely cause of this patient's symptoms? Answer Choices 1 Vestibular neuronitis 2 Ménière's disease 3 Acoustic neuroma 4 Labyrinthitis 5 Benign paroxysmal positional vertigo

Answer: 3 Acoustic neuroma Acoustic neuroma is a benign tumor of the VIII cranial nerve. While these tumors are benign, their growth can lead to sensorineural hearing loss, vertigo, and tinnitus. Because the tumor is slow growing, symptoms typically have a gradual onset. Diagnosis is made by MRI. In vestibular neuronitis, patients present with paroxysmal episodes of vertigo without hearing loss. The patient may also have nystagmus. Ménière's disease also presents with vertigo, sensorineural hearing loss, and tinnitus, but these symptoms are typically episodic in nature, whereas the symptoms of acoustic neuroma are continuous. Labyrinthitis presents with an acute onset of continuous vertigo, hearing loss, and tinnitus. Symptoms gradually improve over several weeks, though hearing may remain impaired following recovery. Acoustic neuroma has a gradual onset, and symptoms will not improve until the tumor is treated. Patients with benign paroxysmal positional vertigo have recurrent episodes of brief vertigo without hearing loss; the vertigo is related to changes in head position. There is no associated hearing loss.

A 23-year-old man presents with a 2-day history of a red, swollen, and painful right knee. The pain and swelling have been progressing steadily since they began. He has also been feeling feverish for the last few hours. He admits to having unprotected sexual intercourse with multiple women over the past few months. On examination, his temperature is 100.4°F (38°C), and BP is 110/70 mm Hg. Cardiovascular, respiratory, and abdominal exam are normal. His right knee demonstrates clinical signs of an effusion, with severe tenderness and surrounding muscle spasm. An aspirate of the joint fluid yields the following findings: WBC count- 60,000/cc, neutrophils 95% No crystals Gram stain is negative Synovial fluid culture: Gram-negative diplococci on Thayer-Martin media Question Considering the probable diagnosis, what is the most appropriate pharmacotherapy? Answer Choices 1 Clindamycin 2 Fluconazole 3 Ceftriaxone 4 Ampicillin 5 Vancomycin

Answer: 3 Ceftriaxone Ceftriaxone is the treatment of choice for gonococci. Several clues in the vignette point towards gonococci as being the causative organism: the Gram-negative diplococci, Thayer-Martin media, and the young sexually active patient all point to gonococci. Clindamycin is used in the treatment of anaerobic infections and mixed infections. Fluconazole is an antifungal; therefore, it does not work on bacteria. Ampicillin is incorrect. While penicillins were effective in the past, the majority of strains of gonococci are now penicillin resistant. Vancomycin is reserved for use against Staph aureus resistant to Methicillin as well as the treatment of C. Diff colitis.

A 42-year-old man presents with a 24-hour history of severe vertigo, tinnitus, hearing loss, nausea, and vomiting intermittently; episodes last at least 30 minutes. After a thorough history and examination, the patient is diagnosed with Ménière's disease. Question What medication is he most likely to be given to treat the acute symptoms? Answer Choices 1Hydrochlorothiazide/triamterene (Dyazide/Maxzide) 2 Meclizine (Antivert) 3 Diazepam (Valium) 4 Acetazolamide (Diamox) 5 Dimenhydrinate (Dramamine)

Answer: 3 Diazepam (Valium) Diazepam (Valium) is a benzodiazepine medication that can be given 5-10 mg IV in order to treat an acute attack during a severe episode of Ménière's disease. Other medications that are fast-acting and can effectively treat an acute attack are atropine and transdermal scopolamine. A few second-line choices for acute treatment are droperidol, promethazine (Phenergan), and diphenhydramine (Benadryl). With the exception of the transdermal scopolamine, all of the listed medications are given by IV or IV push in a controlled setting. Hydrochlorothiazide/triamterene (Dyazide/Maxzide) is a combination medication not used for an acute attack, but it is the first-line treatment for maintenance in patients who have had recurrent attacks. The goal of treating patients with Ménière's disease is to prevent the number of disabling spells of vertigo that they experience. Along with a low sodium diet, diuretics are the mainstay of long-term treatment. Patients should be instructed about a low sodium diet and are to restrict their sodium intake to 1500 mg per day. Other lifestyle changes, such as smoking cessation, caffeine restriction, and alcohol restriction, should also be followed when applicable. If the sodium restriction and diuretic are not effective, then patients should be counseled on an even more restrictive diet of 1000 mg sodium per day and have their diuretic dose increased (if not contraindicated) before considering another treatment option. Meclizine (Antivert) is an antihistamine medication that can be used for maintenance and long-term prevention of recurrent attacks. Acetazolamide (Diamox) is another diuretic medication choice that can be used for maintenance and long-term prevention of recurrent attacks. Dimenhydrinate (Dramamine) is another antihistamine medication that can be used for maintenance and long-term prevention of recurrent attacks.

A 58-year-old woman with a past medical history of hypertension, hyperlipidemia, breast cancer, hip fractures, and coronary artery disease is being evaluated for acute-onset severe left-sided pleuritic chest pain over the course of the last 2 hours. The pain is associated with feelings of anxiety, hemoptysis, shortness of breath, and nausea. She "feels warm" but denies chills, palpitations, wheezing, edema, vomiting, abdominal pain, abnormal bowel habits, or dietary intolerances. She admits to a 30 pack-year smoking history but denies drug or alcohol use. Upon physical exam, she is found to be febrile, hypotensive, tachycardic, tachypneic, diaphoretic, and in acute painful distress. There are perioral cyanosis and a pleural friction rub to the left lung fields; the remainder of the exam is normal. Question What is the most appropriate therapeutic intervention for this patient at this time? 1 Indomethacin 2 Doxycycline 3 Heparin 4 Prednisolone 5 Albuterol

Answer: 3 Heparin This patient's presentation is significant for a pulmonary embolism (PE). Predisposing underlying conditions are almost always present; venous thrombosis may result from a generalized hypercoagulable state, venous endothelial injury, or local stasis (Virchow triad). Most commonly, the initial manifestations of pulmonary embolism include an abrupt dyspnea and chest pain. Tachycardia and hypoxia are the most common clinical signs. Associated manifestations include fever, hypotension, cyanosis, pleural friction rub, and findings consistent with pulmonary consolidation. Anticoagulation is the foundation for successful treatment of deep vein thrombosis and PE. Heparin is the best choice for initial treatment of PE, as it achieves effective anticoagulation immediately. The other answer choices can be used to treat other causes of pleuritic chest pain and other symptoms experienced by the patient in this case, such as indomethacin (pain relief), doxycycline (infection), prednisolone (inflammation), and albuterol (asthma).

A 26-year-old African American man with no significant past medical history presents with a history of dyspnea on exertion that occurs after running. The dyspnea is associated with mild substernal chest pain. All symptoms are relieved with rest. He denies fever, chills, cough, wheezing, pleurisy, calf pain, abdominal complaints, peripheral edema, cigarette, drug, or alcohol use, sick contacts, or travel. His physical exam reveals a harsh murmur best heard at the left lower sternal border and an S4 gallop. A bedside electrocardiogram was remarkable for left ventricular hypertrophy and septal Q waves in the inferolateral leads. An echocardiogram noted asymmetric LVH, anterior motion of the mitral valve during systole, a small and hypercontractile LV, and delayed relaxation and filling of the LV during diastole. The septum was twice the thickness of the posterior wall. Question What is expected to be true regarding the murmur in this patient? 1 It decreases while in an upright posture. 2 It becomes less intense with the Valsalva maneuver. 3 It decreases with squatting. 4 It increases with sustained handgrip. 5 It increases with lying down.

Answer: 3 It decreases with squatting. This patient's presentation is significant for hypertrophic cardiomyopathy (HOCM). The murmur is a loud harsh systolic murmur present along the left sternal border. The gradient and the murmur may be enhanced by maneuvers that decrease ventricular volume, such as an upright posture, standing, or Valsalva maneuver. It is decreased by increasing ventricular volume or vascular resistance, which occurs with squatting, sustained handgrip, lying down, or straight leg raises. These maneuvers help differentiate the murmur of HOCM from that of aortic stenosis, since in HOCM, reducing the left ventricular volume increases obstruction and the murmur intensity; whereas, in valvular aortic stenosis, reducing the stroke volume across the valve decreases the murmur

You are evaluating a 26-year-old man; he is suspected of being infertile. His past medical history is unremarkable. On examination, you note he is 6'4"; he has mild gynecomastia, sparse body hair, and small soft testes. Question What is the most likely diagnosis? Answer Choices 1 Cystic fibrosis 2 Turner syndrome 3 Klinefelter syndrome 4 Fragile X syndrome 5 Hypospadias

Answer: 3 Klinefelter syndrome Klinefelter syndrome is a sex chromosome disorder due to an additional X chromosome on a male karyotype: 47, XXY. Patients present either as adolescent boys who fail to progress through puberty or as adult males suspected of having primary infertility. Low androgen levels lead to gynecoid features, including a female pattern of body hair and gynecomastia. The testes are typically small and soft, and libido also may be reduced. Cystic fibrosis can cause male infertility due to absent development of the vas deferens. Tall body habitus, gynecomastia, and small testes are not typical of cystic fibrosis. Recurrent pulmonary infection and intestinal malabsorption are also features of cystic fibrosis; these are lacking in the described case. Turner syndrome is noted by a 45, X karyotype, short stature, webbed neck, and primary amenorrhea in women. Fragile X syndrome is due to a trinucleotide repeat expansion in the fragile X gene in boys/men. It is also characterized by learning difficulties and, in more severe cases, overt cognitive disability. The testes are typically enlarged. Hypospadias is a developmental problem of the penis where the urethral meatus is proximally placed. Most cases are sporadic, and fertility can be affected, but gynecomastia, altered body hair distribution, and small testes are not typically associated with hypospadias.

A 65-year-old African American man with a past medical history of hypertension, hyperlipidemia, and diabetes experiences substernal chest pain while shoveling snow. The patient says the pain started after 10 minutes of shoveling wet snow and eventually resolved after he sat down and rested. The patient described the pain as a "heaviness" that did not radiate to any other part of his body and as a 4 or 5/10 on a subjective pain scale, and he experienced this discomfort for approximately 1-2 minutes total. The patient's current vital signs are blood pressure 168/98 mm Hg, pulse 92, and respirations 16. Question What drug would be the best choice to rapidly reduce the patient's chest pain in a future similar situation? Answer Choices 1 Aspirin 81 mg PO 2 Metoprolol 50 mg PO 3 Nitroglycerin 0.4 mg SL 4 Simvastatin 10 mg PO 5 Lisinopril 5 mg PO

Answer: 3 Nitroglycerin 0.4 mg SL This patient likely has underlying ischemic heart disease and is experiencing symptoms of acute angina while exerting himself by shoveling snow. If this patient's chest pain is cardiac-related, the medication that will offer the fastest resolution is nitroglycerin. For patients who have angina, nitroglycerin is used in either a sublingual form or spray form at the dose of 0.4 mg. Nitroglycerin is a vasodilator and is used to treat heart conditions, such as angina and chronic heart failure. Aspirin has many uses, but it is most commonly used for minor aches and pains, as an antipyretic, and as an anti-inflammatory agent. Aspirin also has an antiplatelet effect; it has been established that giving aspirin immediately after a heart attack can reduce the risk of another heart attack or of the death of cardiac tissue. While aspirin is used by many general practitioners as a means to reduce stroke and heart attack in patients with risk factors, it is not the best choice to reduce immediate chest pain due to angina. Metoprolol is a selective β1 receptor blocker used in the treatment of several diseases of the cardiovascular system, including hypertension, angina, acute myocardial infarction, supraventricular tachycardia, ventricular tachycardia, congestive heart failure, and prevention of migraine headaches. While metoprolol is used in patients with angina, it is not the fastest or primary drug to reduce the pain of an acute angina attack over nitroglycerin. Statins (HMG-CoA reductase inhibitors) are a class of drugs used to lower cholesterol levels by inhibiting the enzyme HMG-CoA reductase. The primary use of simvastatin is for the treatment of dyslipidemia and the prevention of cardiovascular disease. Lisinopril is a drug of the angiotensin-converting enzyme (ACE) inhibitor class primarily used in the treatment of hypertension, congestive heart failure, and heart attacks, as well as in prevention of renal and retinal complications of diabetes.

A 62-year-old man presents with a 2-month history of worsening fatigue and shortness of breath. He has a past medical history of emphysema attributable to his 85 pack-year cigarette smoking history. The patient complains of nearly passing out while climbing the stairs in his house. He tells you that he feels like his heart races. He reports chronic shortness of breath and cough, but he now he feels like his dyspnea is dramatically worse; he can no longer sleep in his bed. He has been trying to sleep propped up in a chair at night. He is also experiencing fatigue. He has gained about 15 pounds, and he notes that he can no longer lace up his shoes. He denies fever, chills, and chest pain. His cough produces some mucus, but no hemoptysis. His vitals are shown in the table. Weight 212 lb Height 69" Body mass index (BMI) 31.3 Pulse 108 Blood pressure 140/88 mm Hg Temperature 98.2°F Pulse oximetry 88% On physical exam, you see a man in mild respiratory distress; he is sitting upright and leaning forward, and he uses accessory respiratory muscles for breathing. The exam is significant for reduced air movement and mild rales bilaterally in the lungs, distended neck veins, mild tachycardia with prominent P2, lower extremity edema, and right upper quadrant abdominal tenderness with hepatomegaly. Question Once this patient's current condition is stabilized, what health maintenance recommendation is most appropriate for this patient based on the most likely diagnosis? Answer Choices 1 Check blood pressure every 1-2 years 2 Colonoscopy 3 Pneumococcal polysaccharide vaccination 4 Prostate-specific antigen (PSA) testing 5 Weight loss to ideal body weight

Answer: 3 Pneumococcal polysaccharide vaccination Normally recommended for all individuals 65 and over, the pneumococcal polysaccharide vaccination is recommended for younger patients who have chronic lung diseases, such as this patient. Blood pressure screening of adults is recommended by multiple organizations. A recommended interval is every 2 years for those with normal blood pressures and every year for those with mildly elevated blood pressures (systolic 120-139 mm Hg and/or diastolic 80-89 mm Hg). This patient has a systolic blood pressure of 140 mm Hg and other cardiovascular complications, however, so he should be monitored more frequently. If high blood pressure remains elevated, he should be treated. Colonoscopy is a colon cancer screening technique routinely recommended for all men and women 50 and over; it may be recommended earlier if a family history of colon cancer is present. If he has not yet had one, this patient should undergo a screening colonoscopy, but this does not directly relate to his presenting condition (emphysema/chronic obstructive pulmonary disease, progressed into cor pulmonale). Preventive services are most effective when tailored to the patient's specific needs. In the patient's case, a pneumococcal vaccination is a higher priority preventive service than colonoscopy. Prostate-specific antigen (PSA) testing for prostate cancer has been controversial. Currently, the US Preventive Services Task Force is neither for nor against PSA screening. Given his severe pulmonary disease, PSA testing is not a high priority for this patient. This patient's current BMI is over 30, indicating obesity, but the patient's history reveals very recent weight gain attributable to fluid retention. Before any aggressive weight loss is recommended, the edema should be addressed through sodium and fluid restriction and most likely diuretic medications. Furthermore, aggressive physical activity for weight loss would likely exacerbate his condition, so it is not recommended.

Police officers found a 27-year-old man walking aimlessly and shouting the names of former presidents. Urine toxicology is negative, and the man appears to be oriented with respect to person, place, and time. He has had 5 similar admissions over the past year. Attempts to interview the patient are fruitless; he is easily derailed from his train of thought. A phone call to a friend listed in the chart provides the additional information that the man is homeless and unable to care for himself. Question This patient's signs and symptoms are characteristic of what pathology? Answer Choices 1 Schizoaffective disorder 2 Schizoid personality disorder 3 Schizophrenia 4 Schizophreniform disorder 5 Schizotypal personality disorder

Answer: 3 Schizophrenia The patient is suffering from schizophrenia. The key to the diagnosis of psychosis is that there has been a marked decline in the level of functioning (the man is homeless and cannot care for himself). Although hallucinations or delusions are not mentioned in the case history, the presence of disorganized speech, grossly disorganized behavior, and the duration of symptoms (>6 months) suggest a diagnosis of schizophrenia. In schizoaffective disorder, alterations in mood are present during a substantial portion of the illness. Although schizoid personality disorder produces detachment from social relationships and is characterized by restriction of emotional expression, it is not accompanied by a marked decline in occupational functioning. Schizophreniform disorder is characterized by schizophrenia-like symptoms, but the duration of symptoms is, by definition, >6 months. Schizotypal personality disorder is characterized by eccentricities of behavior, odd beliefs or magical thinking, and difficulties with social and interpersonal relationships. Unlike schizophrenia, schizotypal personality disorder is not characterized by a formal thought disorder.

A 15-year-old boy presents with a 1-week history of malaise, a low-grade fever, and a sore throat. On exam, you note pharyngeal erythema, scant yellow exudates, and enlarged posterior cervical lymph nodes. There are no current signs of airway compromise. There is no skin eruption noted, and the rapid strep screen is negative. Question What other physical examination finding is usually present with this condition? Answer Choices 1Pericarditis 2 Pancreatitis 3 Splenomegaly 4 Arthralgia 5 Cullen sign

Answer: 3 Splenomeagaly The clinical picture is suggestive of Epstein-Barr virus infection/infectious mononucleosis. Roughly 50% of patients will have an enlarged spleen. Pericarditis is not a symptom of infectious mononucleosis. Epstein-Barr virus does not affect the pancreas. Arthralgia is a complication of Cytomegalovirus (CMV) infection. Cullen sign is a blue discoloration of the umbilical area and is associated with pancreatitis.

A 72-year-old man with a history of poorly controlled HTN and previous myocardial infarction presents with a nocturnal cough, bilateral ankle swelling, and dyspnea on exertion. He denies any fever, chills, URI symptoms, chest pain, headache, N/V, diaphoresis, or syncope. He further denies smoking, alcohol, or drug use. Physical exam reveals bipedal edema and bibasilar crackles. A chest X-ray is remarkable for enlargement of the cardiac silhouette and interstitial infiltrates, while EKG analysis indicates deep S waves in lead V1 and tall R waves in lead V5. Question What is the most appropriate next step in the evaluation and management of this patient? Answer Choices 1Biopsy of the endocardium 2 Chest CT scan with IV contrast 3 Transthoracic echocardiography 4 Bronchoalveolar lavage 5 Coronary angiography

Answer: 3 Transthoracic echocardiography This patient has clinical manifestations and initial diagnostic test results suggestive of congestive heart failure. The most useful test is an echocardiogram, which is a noninvasive and ionizing-free imaging modality that reveals the size and function of both ventricles and atria. It can also be used to evaluate for the presence of pericardial effusions, valvular abnormalities, intracardiac shunts, and segmental wall-motion abnormalities. An endocardial biopsy may aid in the diagnosis of restrictive cardiomyopathy and constrictive pericarditis, but it is invasive. Chest CT scanning offers the superior delineation of anatomic features to that of plain radiographs; the use of contrast serves to distinguish vascular from nonvascular structures. It does not, however, measure cardiac function. Coronary angiography measures left ventricle ejection fraction and identifies regional wall motion, but it is an invasive procedure and should follow an echocardiogram. Bronchoalveolar lavage is indicated in the bronchoscopic investigation of infiltrative, malignant, occupational, and infectious lung diseases.

A 32-year-old woman with no significant past medical history presents with a 4-day history of vaginal discharge. She describes the vaginal discharge as thin, fairly uniform in its consistency, and of a light grayish color. Her last sexual intercourse encounter was 1 week ago; she admits that it was unprotected. She is not in a monogamous relationship. She denies any fever, chills, swollen glands, dysuria, hematuria, urinary frequency, dyspareunia, or back pain. She further denies any vulvar or vaginal pruritus. Physical exam is significant for a pungent ammonia-like scent with an associated thin gray-white vaginal exudate, but it is otherwise unremarkable. Microscopic evaluation of the vaginal exudates is remarkable for the presence of clue cells. Question What additional laboratory finding would be expected in this patient? Answer Choices 1 Candida spores and hyphae on vaginal wet mount 2 KOH whiff test negative 3 Vaginal pH of 5.2 4 Numerous white blood cells within the vaginal exudate 5 Red blood cell casts upon urinalysis

Answer: 3 Vaginal pH of 5.2 This patient's signs and symptoms are characteristic of bacterial vaginosis. While the infection is polymicrobial in etiology, an overabundance of Gardnerella vaginalis and other anaerobes are causative. Increased malodorous discharge without pruritus, dysuria, or inflammatory changes is typical for the infection. White blood cells are not numerous when vaginal exudate is examined microscopically. This is not a sexually transmitted infection. Clinical criteria for diagnoses include homogeneous white, noninflammatory discharge microscopic presence of > 20% clue cells vaginal discharge with pH greater than 4.5 a fishy odor with or without addition of 10% potassium hydroxide (KOH) Furthermore, Gram staining will reveal small, non-motile, non-encapsulated, and pleomorphic rods. Candida spores and hyphae on a vaginal wet mount would be consistent with a yeast infection, rather than with BV. Red blood cell casts are not typical of bacterial vaginitis; their presence should suggest acute glomerulonephritis, lupus nephritis, SBE, Goodpasture disease, aftermath of streptococcal infection (poststreptococcal glomerulonephritis), vasculitis, or malignant hypertension.

A 32-year-old woman presents with a 1-month history of bleeding gums when brushing her teeth. She also reports that her wounds are taking longer than usual to heal. She is a stay-at-home mother and is breastfeeding her 6-month-old twins. On examination, you note multiple splinter hemorrhages on her nails and ecchymoses over her lower limbs. Question What is the most likely diagnosis? Answer Choices 1Vitamin A deficiency 2 Pyridoxine deficiency 3 Vitamin C deficiency 4 Niacin deficiency 5 Vitamin E deficiency

Answer: 3 Vitamin C deficiency In cases of vitamin C (or ascorbic acid) deficiency, patients can present with bleeding tendencies (as a result of weakened capillaries) and impaired wound healing due to impaired formation of connective tissue. On examination, the gums may be swollen and friable; the teeth may be loose. There may also be multiple splinter hemorrhages on the nails and ecchymoses, especially over the lower limbs. Causes include inadequate dietary intake and certain conditions such as pregnancy and lactation that increase vitamin C requirements. Dietary sources of vitamin C include citrus fruits, such as oranges, lemons, and tangerines, as well as tomatoes and potatoes. In cases of vitamin A deficiency, patients can present with the inability to see well in dim light or night blindness. There may also be conjunctival and corneal xerosis, as well as pericorneal and corneal opacities, and Bitot's spots. Bitot's spots are a collection of keratin appearing as triangular foamy spots on the conjunctiva. Patients may also have xeroderma, hyperkeratotic skin lesions, and increased susceptibility to infections. Causes include inadequate dietary intake and malabsorption. Dietary sources of vitamin A include fish, liver, egg yolk, butter, cream, dark green leafy vegetables, as well as yellow fruits and vegetables. In cases of pyridoxine or (vitamin B6 deficiency), patients can present with peripheral neuropathy, seborrheic dermatosis, glossitis, and cheilosis. Laboratory investigations reveal anemia with lymphopenia. Causes include malabsorption as well as medications, such as isoniazid and penicillamine. Dietary sources of vitamin B6 include liver, legumes, whole grain cereals, and meats. Niacin deficiency causes pellagra, which is characterized by: Symmetrical dermatitis, usually on parts of the body exposed to sunlight Scarlet glossitis and stomatitis Diarrhea Mental aberrations, such as memory impairment, depression, and dementia These may appear alone or in combination. Causes include inadequate dietary intake, especially in patients with corn-based diets or alcoholism. Dietary sources include legumes, yeast, meat, and enriched cereal products. Vitamin E deficiency may cause a hemolytic anemia in premature infants. Laboratory investigations reveal low plasma tocopherol levels, a low hemoglobin level, reticulocytosis, hyperbilirubinemia, and creatinuria. Causes of vitamin E deficiency in premature infants include limited placental transfer of vitamin E and the resultant low levels at birth combined with its relative deficiency in the infant diet. Dietary sources for older children and adults include wheat germ, vegetable oils, egg yolk, and leafy vegetables.

A 28-year-old man presents with rectal bleeding. The patient had noticed blood with bowel movements 3 times. The blood is described as bright red in color and small in amount. He also complains of rectal pain, especially with passing hard stools. He has tried some over-the-counter hemorrhoid creams without relief. The patient admits episodic constipation. He denies dark, tarry stools, easy bruising, and prior episodes of rectal bleeding. He has not noticed blood in his urine or with brushing his teeth. He denies nausea, vomiting, diarrhea, fevers, and weight loss. He has no known medical conditions. Family history is negative for gastrointestinal disorders. Social history reveals he is in a heterosexual relationship and denies anal intercourse. On physical exam, abdomen is normal. The anus has no visible protrusions or rash; however, there is a very small, erythematous and tender area that appears like a "paper cut" or crack in the skin. The patient experiences pain with digital rectal exam (DRE). No masses are noted in the rectal vault. Question What is the most appropriate prescription treatment for this patient's current condition? Answer Choices 1 Bacitracin ointment 2 Minoxidil topical 3 Nitroglycerin ointment 4 Nystatin topical 5 Tretinoin topical

Answer: 3 nitroglycerin ointment This patient most likely has an anal fissure. Fissures are small, painful lesions in which the skin has a "cut" or "torn" appearance. The fissure most likely occurred secondary to anal sphincter stretching and tearing with passing of large, firm bowel movements. Fissures can be associated with a small amount of bright red rectal bleeding. Intra-anal use of nitroglycerin ointment is a first-line treatment for anal fissures. The nitroglycerin improves perfusion of the surrounding tissue and lowers elevated resting pressures. Bacitracin ointment is a topical antimicrobial, used for mild, superficial skin infections and is widely available over-the-counter. It would not treat anal fissures. Minoxidil topical is a treatment used to stimulate hair growth for alopecia. Its use is not appropriate for this patient. Nystatin topical is an antifungal, commonly used for treatment of candida infections, such as thrush. It would not treat anal fissures. Tretinoin topical is a retinoid medication used for dermatologic conditions such as acne and wrinkles. It would not treat anal fissures.

Classification of shock is typically categorized into 4 areas of origin. The cause of shock can drastically affect the mode of treatment. If the etiology of a patient's shock is myopathic, mechanical, or arrhythmic, then this would be considered what type of shock? Answer Choices 1Hypovolemic 2 Distributive 3 Obstructive 4 Cardiogenic 5 Septic

Answer: 4 Cardiogenic Cardiogenic is the correct response. Early identification of shock is a necessity to help improve any patient's chance of survival and recovery. It is also critical in regard to reversing the cause of the shock and initiating early resuscitation efforts. Early on, patients suffering from shock may have only a few or very subtle symptoms, such as tachypnea, tachycardia, hyperthermia, hypothermia, weak or bounding peripheral pulses, delayed capillary refill, and/or pale or cool skin. Decreased mental status, weak or absent central pulses, central cyanosis, hypotension, and bradycardia are ominous and later signs indicating the shock has progressed. Depending on its etiology, shock is generally classified into 4 major categories: cardiogenic, hypovolemic, distributive, and obstructive. Cardiogenic shock occurs as a consequence of cardiac pump failure. Myopathic, mechanical, and/or arrhythmic issues lead to cardiogenic shock. Hypovolemic shock results from a multitude of different causes, including but not limited to hemorrhagic or non-hemorrhagic causes, such as ruptured ectopic pregnancy, gastrointestinal loss, dehydration, burns, diabetic ketoacidosis, and/or third-space fluid loss. Whatever the cause, there is typically an acute loss of 15-20% of circulating blood volume, which leads to lack of blood to pump throughout the circulatory system. Loss of vascular tone and enlargement of the vascular compartment is the origin of distributive shock. Distributive shock is caused by sepsis, adrenal crisis, neurogenic, and/or anaphylactic issues. Obstructive shock is caused by obstruction of flow due to impaired cardiac filling as well as excessive afterload. Obstructive shock results from pulmonary embolism, tension pneumothorax, cardiac tamponade, and/or constrictive pericarditis.

A 67-year-old man with a prior history of endocarditis and a prosthetic mitral valve presents requesting antibiotic prophylaxis for a dental procedure that entails the extraction of one of his molars. He is able to speak and swallow normally and is presently asymptomatic. His physical exam is remarkable only for a "loudly-clicking S1" in the mitral position. He has a known allergy to penicillin that results in urticaria and angioedema upon exposure. Question What is the best intervention concerning this patient? Answer Choices 1 Amoxicillin 2 grams PO within 1 hour before the procedure 2 No antibiotic prophylaxis for the dental procedure is necessary 3 Cephalexin 2 grams PO 1 hour before the procedure 4 Clarithromycin 500 mg PO 1 hour before the procedure 5 Clindamycin 600 mg IV or IM 1 hour before the procedure

Answer: 4 Clarithromycin 500 mg PO 1 hour before the procedure Clarithromycin 500 mg PO 1 hour before the procedure is the correct response. The American Heart Association and the European Society of Cardiology currently recommend prophylactic antibiotics only for those patients at highest risk for severe morbidity or death from endocarditis. Cardiac conditions in which prophylactic antibiotics are recommended include high-risk cardiac lesions such as prosthetic heart valves; prior endocarditis; unrepaired cyanotic congenital heart disease including palliative shunts or conduits; completely repaired congenital heart defects during the 6 months after repair; incompletely repaired congenital heart disease with residual defects adjacent to prosthetic material; and valvulopathy developing after cardiac transplantation. This patient meets 2 of the identified high-risk cardiac lesions and should receive prophylaxis. His allergic history precludes the use of amoxicillin and cephalexin, while desensitization is unnecessary given available alternatives. The patient speaks and swallows without difficulty, making clarithromycin the most suitable agent. Clindamycin would also be appropriate for this patient, but not via IV or IM.

A 66-year-old man presents with a 2-month history of fatigue. He reports that he has recently joined Alcoholics Anonymous. On examination, he is malnourished and pale, but his neurological examination is essentially normal. A peripheral blood smear reveals macrocytic red cells. Question What is the most likely diagnosis? Answer Choices 1Iodine deficiency 2 Zinc deficiency 3 Copper deficiency 4 Folic acid deficiency 5 Cobalamin deficiency

Answer: 4 Folic acid deficiency Patients with folic acid deficiency present with symptoms of anemia (e.g., fatigue, weakness, syncope). On examination, patients are pale. Laboratory analysis reveals macrocytic red blood cells and hemoglobin <12 g/dL. Causes include malabsorption, inadequate intake by chronic alcoholics and malnourished individuals, increased demand during pregnancy, and chronic hemolytic anemias. Dietary sources of folic acid include green leafy vegetables, liver, and yeast. Patients with vitamin B12 (cobalamin) deficiency can present with symptoms and signs of anemia (e.g., fatigue, lightheadedness, syncope, pallor); there are also neurological signs and symptoms (e.g., ataxia, paresthesias, diminished proprioceptive and vibratory sensations in the lower limbs). Laboratory analysis reveals macrocytic red blood cells. Causes include pernicious anemia, blind loop syndrome, fish tapeworm infestation, and vegetarian diet. Dietary sources of vitamin B12 include liver, beef, eggs, and milk. In cases of iodine deficiency, patients can present with an anterior neck swelling that rises with deglutition. This is usually a colloid goiter; the thyroid hypertrophies as it tries to concentrate iodide in itself. Though most of these patients are euthyroid, some may develop hypothyroidism. Cretinism can develop in infants as a result of iodine deficiency; impaired brain development and fetal growth can also develop. Causes include inadequate dietary intake. Dietary sources of iodine include iodized table salt, seafood, eggs, and dairy products. In cases of zinc deficiency, patients can present with hypogeusia (decreased taste sensation), anorexia, delayed sexual maturation, night blindness, and hair loss. On examination, they have alopecia, growth retardation, delayed sexual maturation, and hypogonadism. Laboratory analysis may reveal hypospermia. Causes include malabsorption states, alcoholism, and prolonged parenteral nutrition. Dietary sources of zinc include beef, liver, eggs, and oysters. Menkes syndrome is an inherited copper deficiency caused by mutations in an X-linked gene. It occurs in male infants and is characterized by intellectual disability, kinky hair, hypopigmentation, and vascular aneurysms. Laboratory analysis reveals hypocupremia and decreased circulating ceruloplasmin. Other causes of copper deficiency include severe malabsorption, infants with persistent diarrhea fed on milk diets, copper-free total parenteral nutrition, and excess intake of zinc salt dietary supplements, which can interfere with copper absorption. Dietary sources of copper include organ meats, oysters, nuts, dried legumes, and whole grain cereals.

A 42-year-old female teacher presents with a hand tremor; the tremor has been present for several years, but it is getting more bothersome. The patient would like treatment for her tremor. It appears to be present only in her right hand, and she tells you that it "comes and goes." It is especially embarrassing for her because she writes on the board in front of her students. She denies pain, motor weakness, and abnormal sensation in the right hand. She reports that her father's hand used to "shake" when he drank his coffee. She otherwise feels well, and she denies other symptoms. She has not noticed memory problems, incoordination, or balance problems. Her past medical history is unremarkable. She had 3 uncomplicated pregnancies and deliveries; she has no chronic medical conditions. Except for a mild tremor on the patient's right hand when held extended, the physical exam is unremarkable. Her neurological exam is otherwise normal. Question What is the most appropriate intervention for this patient's condition? Answer Choices 1Advise the patient her condition is benign and no treatments are available. 2 Initiate carbidopa/levodopa. 3 Initiate methylphenidate. 4 Initiate propranolol. 5 Refer her for physical therapy.

Answer: 4 Initiate propranolol. Based on her history and exam, this patient has a benign essential (familial) tremor, for which it is appropriate to initiate propranolol. Propranolol is a non-selective beta blocker. This patient has no contraindications to the therapy; she is troubled by the worsening tremor, which interferes with her daily activities. Essential tremor is indeed a benign condition. When the tremor is mild and does not cause embarrassment or interfere with the patient's activities, it would be reasonable to avoid medications, but it is inappropriate to tell the patient that no treatments exist. Especially since this patient desires treatment. Carbidopa/levodopa would be an appropriate medication for an individual with tremor due to Parkinson's disease, but this patient has no other neurological complaints and no history to suggest she currently has Parkinson's disease. Methylphenidate is a stimulant typically used for the treatment of attention deficit hyperactivity disorder. Not only would it be ineffective in this patient's case, it would also likely make her tremor worse. Physical therapy has not been proven to have a role in treating essential tremor. It would not be helpful in this case.

A 30-year-old Caucasian man presents with a 3-day history of fever with chills and severe weakness. There are no other complaints. The patient has had multiple sex partners in the past. He also gives a history of travel to South America and consumption of street food while working there 1 month before presentation. He admits to intravenous drug abuse and cocaine abuse (snorting) in his early 20s. He often ventures out into the woods and has been bitten by several insects in the recent past. Abdominal exam reveals mild hepatomegaly. You send for routine lab investigations, including CBC, comprehensive panel, and serology of HIV and Hepatitis B and C. Liver enzymes are elevated and anti-HCV comes back positive. Anti-HIV and HBV are negative. Question How did this patient most likely acquire the hepatitis C infection? Answer Choices 1 Sex with multiple partners 2 Consumption of street food 3 Cocaine abuse 4 Intravenous drug abuse 5 Tick bite

Answer: 4 Intravenous drug abuse The most likely means of transmission of hepatitis C virus (HCV) is by exposure to infected blood, and IV drug abuse is the most common source. In the United States, intravenous drug abuse is the most common mode of transmission of hepatitis C. Other modes of transmission include receiving blood or blood products, organs, needlestick injuries in healthcare, and vertical transmission from an infected mother. Other less frequent modes include sexual transmission from an infected partner, sharing personal items such as razors (which may have blood on them), and from healthcare procedures such as dental procedures or injections. There is limited information on whether cocaine snorting carries an additional risk of hepatitis C transmission. Hepatitis A, not hepatitis C, is transmitted by contaminated food from regions endemic for hepatitis A, such as South America, Asia, and Africa. Tick bites are not known to transmit HCV.

A 30-year-old woman presents for routine analysis of cholesterol levels. The results show plasma cholesterol levels of 300 mg/100 mL. You prescribe the drug simvastatin (Zocor). She is reluctant to take drugs to treat her hypercholesterolemia. After further discussion, she agrees to take a vitamin to treat the elevated cholesterol. She also has questions concerning familial hypercholesterolemia. Question What vitamin is effective in reducing circulating cholesterol levels when given in pharmacological doses? Answer Choices 1Riboflavin 2 Thiamin 3 Vitamin K 4 Nicotinic acid 5 Pantothenic acid

Answer: 4 Nicotinic acid When given at pharmacological doses of 2-4 g/day, nicotinic acid has many effects unrelated to its normal vitamin function. Nicotinic acid (niacin) is normally found in the coenzyme nicotinamide adenine dinucleotide (NADH). In high doses, nicotinic acid decreases the mobilization of fatty acids in adipose tissue, decreases the circulating levels of cholesterol and lipoproteins, and increases serum glucose and uric acid. These effects are not fully understood, but they can be partially explained by the effect of nicotinic acid on cAMP levels. Nicotinamide does not show these effects. Continued use of nicotinic acid can result in liver damage. The other vitamins listed above do not affect cholesterol levels. Thiamine is involved in many cell- and nerve-related activities. Riboflavin is a component that plays a key role in energy-producing respiratory pathways. Pantothenic acid plays a crucial role in the formation and degradation of many peptide hormones, including ACTH. Vitamin K functions as a part of the clotting and coagulation pathway.

A 56-year-old woman presents with severe left-sided flank pain. She has been treated for kidney stones 2 times in the past year. Review of systems is positive for polydipsia, fatigue, overall weakness, and depression. She denies fever, hematuria, vaginal discharge, constipation, or diarrhea. Her past medical history is significant for hypertension, which is controlled with hydrochlorothiazide. Her serum laboratory values are listed in the table. Serum Reference range Result Blood urea nitrogen 8-20 mg/dL 19 mg/dL Creatinine 0.6 -1.2 mg/dL 1.1 mg/dL Glucose 60-110 mg/dL 82 mg/dL Phosphorus 2.5-4.5 mg/dL 1.9 mg/dL Ionized calcium 4.6-5.3 mg/dL 5.9 mg/dL PTH, intact 11-54 pg/mL 87 pg/mL Question What is the underlying diagnosis contributing to her recurrent nephrolithiasis? 1 Chronic kidney disease 2 Diabetes mellitus 3 Metastatic breast cancer 4 Parathyroid adenoma 5 Thiazide diuretic overuse

Answer: 4 Parathyroid adenoma Parathyroid adenoma is the correct response. The patient has hypercalcemia that is contributing to her recurrent nephrolithiasis. Patients with hypercalcemia may be asymptomatic or present with "bones, stones, abdominal groans, psychic moans, with fatigue overtones." The most common causes of hypercalcemia are primary hyperparathyroidism and malignancy, accounting for over 90% of cases of hypercalcemia. To determine the cause of hypercalcemia, serum ionized calcium, phosphate, intact parathyroid hormone (iPTH), and urine calcium levels should be evaluated. If malignancy is suspected, appropriate workup should be performed. This patient has elevated ionized calcium, elevated calcium in her urine, and hypophosphatemia, with elevated parathyroid hormone levels consistent with primary hyperparathyroidism. Primary hyperparathyroidism is caused by unregulated PTH secretion. In 80% of cases, it is caused by a single parathyroid adenoma. It is more common in patients over 50 and affects women more than men. Laboratory results show increased serum and urine calcium, increased iPTH, and decreased or normal serum phosphorous. Imaging is not necessary, but it may be used in a preoperative mapping of the parathyroid glands. Medical treatment includes fluids, bisphosphonates, or cinacalcet. Symptomatic patients can be cured with a parathyroidectomy. Calcium and vitamin D supplementation may be required for a reactive secondary hyperparathyroidism occurring after surgery. Patients with chronic kidney disease will have increased blood urea nitrogen and creatinine levels. They develop a secondary hyperparathyroidism due to decreased vitamin D metabolism, resulting in decreased serum calcium levels and increased PTH levels with increased serum phosphorus levels. Patients with hypercalcemia can develop polyuria and polydipsia due to nephrotic diabetes insipidus. These symptoms can be similar to diabetes mellitus (DM). In DM, the serum calcium, phosphorus, and parathyroid levels will be normal with elevated blood glucose levels. Thiazide diuretic use can result in hypercalcemia, but the urine calcium level will be low. PTH and phosphorus levels will be normal. Malignancy should be evaluated as a possible cause of hypercalcemia. If the patient has not undergone routine screenings, they should be done. The PTH level will be low in a patient with malignancy-related hypercalcemia.

A 12-year-old Caucasian girl presents with a sore throat. The onset of symptoms was about 24 hours ago. The patient experiences pain in her throat, especially with talking or swallowing. She is also very fatigued because throat pain prevented her from sleeping last night. Throat lozenges have not been helpful. She attends public school with several other classmates who have been out sick recently. The patient denies nasal congestion, rhinorrhea, and cough and is unsure of fevers. This patient has no chronic medical conditions, takes no medications, and has no known drug allergies. Her vitals are: Weight 92 lb Height 56: Pulse 95 Blood pressure 102/60 Temperature 99.2°F/37.3°C On physical exam, the patient appears slightly ill and fatigued. Her HEENT exam is positive for bilateral cervical lymphadenopathy and inflamed posterior oropharynx without exudate. She does have normal range of motion of the neck without eliciting pain. Her heart, lung, and abdominal exams are normal. No other lymph nodes are palpable. Question What is the most appropriate next step in the care of this patient? Answer Choices 1 Excuse the patient from school until symptoms have fully resolved. 2 Initiate amoxicillin. 3 Order complete blood count (CBC) with differential. 4 Perform rapid antigen testing for group A streptococcus. 5 Recommend fluids, rest, and aspirin for management of pain and fever.

Answer: 4 Perform rapid antigen testing for group A streptococcus. The most appropriate step in the management of this patient with acute pharyngitis is to perform rapid antigen testing for group A streptococcus. While viral causes of pharyngitis are extremely common, management should focus on identifying those at risk for group A beta-hemolytic streptococcal infections to prevent compilations such as rheumatic fever and glomerulonephritis. This patient presents with acute pharyngitis, no cough, and cervical lymphadenopathy, which all support a likely diagnosis of streptococcal origin. Additional indicators would be temperature over 38°C and tonsillar exudate. Excusing the patient from school while symptoms are still present may limit the spread of the infection and allows the patient to rest, but this does not address the possible need for antibacterial treatment and would leave the patient at risk for developing complications if she indeed has group A streptococcal pharyngitis. Initiation of amoxicillin may be appropriate management once group A streptococcal pharyngitis is confirmed. (Penicillin has been the standard primary regiment.) However, if this patient's pharyngitis is not found to be caused by streptococcus (and therefore, most likely viral), use of antibiotics is unnecessary and only puts the patient at risk for adverse effects of the medication. In fact, if this patient's pharyngitis is caused by infectious mononucleosis, use of amoxicillin can precipitate a skin rash. A complete blood count (CBC) will likely show some characteristic changes in patients affected by acute pharyngitis. For example, the total white blood cell (WBC) count is likely to be elevated. Depending on the potential cause of the pharyngitis, the neutrophils and monocytes may be elevated for bacterial and viral causes, respectively. The CBC changes are rather unspecific, however, especially early in the infection. It makes more clinical sense to directly swab the patient's throat and get a result within minutes rather than to subject a 12-year-old to a blood draw with a typically longer wait time for results. The conservative approach of fluid, rest, and analgesics/antipyretics is reasonable if the patient is not likely to have group A streptococcal pharyngitis. Aspirin is not the best choice for a 12-year-old, though. Because of the link between aspirin use and Reye's syndrome in children, salicylate use is discouraged in children, especially if suffering viral illnesses.

A 28-year-old woman presents with severe right ankle pain. Further questioning reveals the patient is unable to move her ankle. She states that it is exquisitely tender and the pain worsens with movement or when attempting to bear weight. She states that she was immediately unable to take any steps because of how substantially painful the right leg is. During the physical examination, the patient continues to be unable to bear weight on the injured leg. You note that the right ankle is painful to even light palpation throughout the malleolar zone. The skin is a healthy color and there are no signs of lacerations or tenting. Pulses are palpable. The entire area of the right ankle is swollen and appears dislocated. The left leg, foot, and ankle are unremarkable. Question Given the history and physical examination, what is this patient most likely suffering from? Answer Choices 1Right inversion ankle sprain 2 Right eversion ankle sprain 3 Right ankle strain 4 Right ankle fracture 5 Right ankle hematoma

Answer: 4 Right ankle fracture The correct answer for this patient scenario is a right ankle fracture. Since there is significant swelling bilaterally, both the tibia and the fibula are very highly likely to be fractured. Key features of an ankle fracture include pain, swelling/signs of bruising after the traumatic event, apparent deformity, and the inability to bear weight; this patient fits all of these components. Ottowa Ankle Rules have been found to determine with high sensitivity if an ankle fracture can be ruled out. The rules state an ankle X-ray is only required if: There is any pain in the malleolar zone; and, Any one of the following: Bone tenderness along the distal 6 cm of the posterior edge of the tibia or tip of the medial malleolus, OR Bone tenderness along the distal 6 cm of the posterior edge of the fibula or tip of the lateral malleolus, OR An inability to bear weight both immediately and in the emergency department for four steps. Inversion ankle sprains will present with localized pain and swelling over the lateral aspect of the ankle only. The patient would have difficulty weight bearing and would still attempt to walk, although the patient would most likely limp. Eversion ankle sprains are also termed high ankle sprains. This injury is commonly acquired after the foot being turned out or externally rotated and everted, as during a tackle. Pain associated with this sprain is typically worse than with other sprains. Mild swelling or even an effusion may be present. No dislocation is apparent. Ankle strains involve the stretching or tearing at the junction where the muscle meets the tendon (sprains have to do with stretching or tearing of the ligament). Acute strains will present with pain, muscle spasms, loss of strength, and potentially limited range of motion. Strains take place with the muscle is stretched and suddenly contracts, as is the case in running or jumping. Hematomas typically cause some discomfort or pain, but they rarely cause such severe issues as dislocation of the ankle or the inability to bear weight.

A 3-year-old girl is brought to the clinic by her mother, who tells you that the child has not been eating well over the past month and has developed swelling in the abdomen. On exam, the child has a smooth abdominal mass that is the size of a baseball on the left side. Vital signs reveal a blood pressure of 134/82 mm Hg, temperature of 99.8°F, and respirations of 16 breaths per minute. Urinalysis shows only 1+ red blood cells, and CBC and CMP are within normal limits. Question What is the most likely diagnosis? Answer Choices 1Multicystic dysplastic kidney disease 2 Splenomegaly 3 Lymphoma 4 Wilms tumor 5 Renal cell carcinoma

Answer: 4 Wilms tumor Wilms tumors account for 95% of all urinary tract malignancies in childhood. The median age of diagnosis is 3. These tumors are usually a solitary unilateral mass. Children typically present with an enlarging smooth abdominal mass confined to one side of the abdomen. Other associated symptoms include abdominal pain, fever, hypertension (60%), and hematuria (25%). Multicystic dysplastic kidney disease is more common on the left side and is more common in boys. 60% of unilateral multicystic dysplastic kidneys involute in the first 3 years of life; instead of getting bigger to reveal a palpable mass, these typically get smaller over time in children. Splenomegaly in children is usually indicative of hepatic or hematologic disease. This child's CBC and CMP are within normal limits, making this diagnosis unlikely. Lymphoma often presents with painless, firm lymphadenopathy typically confined to 1 or 2 lymph node areas (usually the supraclavicular and cervical nodes). Mediastinal lymphadenopathy is also a common presentation. It can manifest with coughing and shortness of breath. Abdominal masses are not a common presentation of lymphoma, and CBC is typically abnormal in these patients. Renal cell carcinoma accounts for only 3% of childhood urinary tract malignancies. Wilms tumor is much more likely to be the diagnosis in a child.

A 92-year-old woman with Alzheimer's disease is brought in from her long-term care facility. She was found holding her abdomen and screaming in pain. She is unable to provide any history, but her aide states that the patient was incontinent of liquid stool shortly before being brought to the hospital. On examination, her abdomen is distended; there are no palpable masses. On rectal examination, there are firm feces in the rectal vault. Question What is the best initial treatment? Answer Choices 1Fiber supplementation 2 Decompressive colonoscopy 3 Magnesium citrate 4 Mineral oil enema 5 Colectomy

Answer: 4 mineral oil enema Mineral oil enema is the correct response. The patient is suffering from fecal impaction. Initial treatment requires resolving the impaction by the use of enemas, digital removal, or a combination of the two. This is necessary to avoid the development of large bowel obstruction. Once the patient's obstruction has resolved, fiber may be used to help prevent the development of constipation and further episodes of fecal impaction. Unless the impaction has resolved, fiber supplementation could actually harm the patient A colonoscope would not be able to make it pass the stool involved in the fecal impaction. Magnesium citrate is incorrect. The patient should have therapy aimed at disrupting the fecal impaction. Colectomy is incorrect. Surgery would not be warranted in this patient unless she showed signs of perforation.

A 70-year-old man with a history of hyperlipidemia, hypothyroidism, osteoarthritis, and hypertension presents for a routine evaluation. He denies any complaints today and otherwise has no significant past medical history. His physical examination is remarkable for a pulse rate of 44 beats per minute and a blood pressure of 150/94 mm Hg. An EKG assessment reveals sinus bradycardia and a type II Mobitz heart block. Question What antihypertensive should be avoided in the management of this patient? Answer Choices 1Fosinopril 2 Hydrochlorothiazide 3 Olmesartan 4 Aliskiren 5 Metoprolol

Answer: 5 Metoprolol Metoprolol is contraindicated in sinus bradycardia, second- or third-degree AV block, overt heart failure, or cardiogenic shock. Commonly used medications that cause or contribute to bradycardia do so by enhancing vagal tone (for instance, digitalis), reducing the facilitation of AV conduction that results from sympathetic tone (e.g., blockers and antiarrhythmic agents with blocking properties, such as sotalol and propafenone) or direct action on SA and AV conduction tissue (e.g., verapamil and diltiazem). Simple withdrawal of these medications will reverse bradycardia. Fosinopril (ACE inhibitor), hydrochlorothiazide (diuretic), olmesartan (angiotensin receptor antagonist), and aliskiren (direct rennin inhibitor) will not have inhibitory effects at the AV node and are therefore acceptable considerations in managing this patient's hypertension, given his bradycardia and heart block.

A 32-year-old man presents with pain in his back, buttocks, and posterior thighs for 2 days after lifting a heavy load at work. He denies pain when sleeping unless he rolls over in bed. He also has pain with just standing or sitting still in a chair. He denies any radicular symptoms or bladder or bowel dysfunction. Question What diagnostic tests does this patient require? Answer Choices 1MRI of the LS spine 2 CT of the LS spine 3 Plain radiographs of the LS spine 4 Electromyogram (EMG) 5 No diagnostic tests are required at this time

Answer: 5 No diagnostic tests are required at this time No diagnostic tests are required with an acute lower back pain patient initially. Plain films usually are not helpful to diagnose acute back pain/strain. If the patient has pain at night, at rest, or has a history of significant trauma, however, then radiographs are indicated. MRI is best used for patients with findings suggestive of nerve impingement from a herniated disk. CT, though able to visualize herniated disk material, is best used for boney pathology. EMG is indicated if there is a long-standing nerve impingement.

A 35-year-old man with no significant past medical history has been experiencing progressive, moderately severe right knee pain for the past 3 months. Pain was initially felt only at night, but it is now constant throughout the course of the day for the last several weeks. It is especially severe upon ambulation and during knee ranges of motion, causing him to limp. He denies fever, chills, weight changes, history of gout, sexually transmitted diseases, hip or back pain, recent instrumentation, trauma, or injuries. His physical exam reveals an antalgic gait with limp, limited ranges of motion of the right knee, and a 3 cm diameter firm, tender mass at the distal femur. There is no erythema, crepitus, alignment deformity, or effusion noted. Question What is the next appropriate step in the management of this patient? Answer Choices 1 Order a serum uric acid level and prescribe indomethacin. 2 Wrap the right knee in an elastic bandage and prescribe physical therapy. 3 Perform a joint aspiration with microscopic cellular assessment. 4 Obtain a blood culture and begin empiric dicloxacillin. 5 Order an LDH, ALP, and a plain radiograph of the knee.

Answer: 5 Order an LDH, ALP, and a plain radiograph of the knee. This patient's manifestations are most suggestive of osteosarcoma. It is typically seen in patients in their teens and 20s, occurs more frequently in men than in women, and is found in the metaphyseal areas of long bones, with 50% of lesions seen about the knee joint. The distal femur is the most common site, followed by the proximal tibia, and then the proximal humerus. The only blood tests with prognostic significance are lactic dehydrogenase (LDH) and alkaline phosphatase (ALP). Patients with an elevated ALP at diagnosis are more likely to have pulmonary metastases. In patients without metastases, those with an elevated LDH are less likely to do well than are those with a normal LDH. Other laboratory tests include a complete blood cell (CBC), including a differential and platelet level, electrolytes, and liver and renal function tests. The practitioner should also obtain plain films of the suspected lesions in two views. Elevation of the periosteum may appear as the characteristic Codman triangle; in approximately 60% of cases, extension of the tumor through the periosteum may result in a "sunburst" appearance.

A 32-year-old woman with no significant past medical history presents with a 3-month history of right anterior knee pain described as a dull and aching pain that is "right under the kneecap." Provocative activities include bending movements, descending stairs, and performing squatting maneuvers. Pain is relieved during rest. She notes that she loves the outdoors and her hobbies include running and hiking; her symptoms began following a run. She denies a history of falls, prior surgeries or instrumentation, fever, chills, malaise, myalgias, changes in weight, joint swelling, skin changes or rashes, or other joint pains. Her physical exam is normal with the exception of a tender undersurface of the patella, with crepitus upon passive range of motion of the right knee. There is abnormal patellar tracking upon right knee flexion and apprehension of the patient upon passive manipulation of the patella. Additionally, there is a positive patellar grind test. McMurray's, Lachman, the anterior and posterior drawer, Apley's compression and distraction tests, and varus/valgus tests are all negative. There is no joint line tenderness, effusion, or restriction of range of motion of the right knee. Question What is the most likely diagnosis? Answer Choices 1 Right knee meniscal tear 2 Tear of the medial collateral ligament (MCL) 3 Anterior cruciate ligament (ACL) tear 4 Iliotibial band syndrome 5 Patellofemoral pain syndrome

Answer: 5 Patellofemoral pain syndrome This patient exhibits signs and symptoms of patellofemoral pain syndrome, also known as chondromalacia patellae, miserable malalignment syndrome, and runner's knee. This syndrome describes any pain involving the patellofemoral joint. The pain affects any or all of the anterior knee structures, including the medial and lateral aspects of the patella, as well as the quadriceps and patellar tendon insertions. The patella engages the femoral trochlear groove with approximately 30° of knee flexion. Forces on the patellofemoral joint increase up to 3 times body weight as the knee flexes to 90° (e.g., climbing stairs), and 5 times body weight when going into full knee flexion (e.g., squatting). Abnormal patellar tracking during flexion can lead to abnormal articular cartilage wear and pain. When the patient has ligamentous hyperlaxity, the patella can sublux out of the groove, usually laterally. Patellofemoral pain is also associated with muscle strength and flexibility imbalances, as well as altered hip and ankle biomechanics. Patients usually complain of pain in the anterior knee with bending movements and less commonly in full extension. Pain from this condition is localized under the kneecap but can sometimes refer to the posterior knee or over the medial or lateral inferior patella. Symptoms may begin after a trauma or after repetitive physical activity, such as running and jumping. When maltracking, palpable and sometimes audible crepitus can occur. Intra-articular swelling usually does not occur unless there are articular cartilage defects or if osteoarthritis changes develop. Patellar mobility can be assessed by medially and laterally deviating the patella (deviation by one-quarter of the diameter of the kneecap is consider normal; greater than one-half the diameter suggests excessive mobility). The apprehension sign suggests instability of the patellofemoral joint and is positive when the patient becomes apprehensive when the patella is deviated laterally. The patellar grind test is performed by grasping the knee superior to the patella and pushing it downward with the patient supine and the knee extended, thus pushing the patella inferiorly. The patient is asked to contract the quadriceps muscle to oppose this downward translation, with reproduction of pain or grinding being the positive sign for chondromalacia of the patella. Iliotibial band syndrome typically presents with a lateral "snapping" with flexion and extension of the knee. Patients with ACL tears hear an audible "pop" and usually fall down following the injury. They also have acute swelling, difficulty with bearing weight, and instability. Positive Lachman and anterior drawer tests are observed. Medial collateral ligamentous injuries usually occur as a result of a valgus stress to the partially flexed knee. It can also occur with a blow to the lateral leg. The MCL is commonly injured with acute ACL injuries. The patient may have a limited range of motion due to pain, especially during the first 2 weeks following the injury. A positive valgus stress test is expected. Injuries to a meniscus can lead to pain, clicking, and locking sensation. Most meniscus injuries occur with acute injuries (usually in younger patients) or repeated microtrauma, such as squatting or twisting in older patients. Clinical findings include an antalgic gait, difficulty with squatting, catching, or locking of the meniscal fragment, effusion, and joint line tenderness. Patients can usually point out the area of maximal tenderness along the joint line. The McMurray test is usually present.

A 20-year-old female college student presents to the student health center with a 1-week history of daily headaches. She has no significant past medical history. Upon further questioning, she admits to headaches occasionally over the past 2 years, but it is lasting longer this time. She classifies the pain as a 4/10 that is generalized, but is worse in the back of the head. The headaches are not debilitating or throbbing, and she describes it as an annoying pain. Acetaminophen helps somewhat but does not make the headache go away completely. On physical exam, there are no neurologic deficits, vision is 20/20 uncorrected, and vital signs are within normal limits. She does note tenderness upon palpation of the muscles of the head, neck, and shoulders. Question What is the most likely diagnosis? Answer Choices 1 Cluster headache 2 Giant cell arteritis 3 Intracranial mass 4 Migraine headache 5 Tension headache

Answer: 5 Tension Headache Tension headache is the most common type of primary headache disorder. Common complaints include pericranial tenderness, poor concentration, and daily headaches (often vice-like or tight). These can be exacerbated by emotional stress, fatigue, noise, or glare. Cluster headaches affects mostly middle-aged men and typically present with unilateral periorbital pain. Migraine headaches usually present with a lateralized throbbing pain that occurs episodically following its onset in adolescence or early adult life. These headaches can be associated with nausea, vomiting, anorexia, phonophobia, photophobia, and blurred vision. Giant cell arteritis affects mostly elderly patients and can cause headaches that are preceded by myalgia, malaise, anorexia, and weight loss. Loss of vision is a common manifestation. Intracranial masses can also cause headaches that can range from mild to severe. These types of headaches are often accompanied by neurologic deficits.

A 42-year-old woman presents with a 1-year history of hand tremors. She tells you that it occurs in both hands and at times is worse in one hand than the other. It is especially embarrassing for her because, as a teacher, she writes on the board in front of her students. She denies pain, motor weakness, and abnormal sensation in her hands. She reports that her father's hands used to "shake" when he drank his coffee. Otherwise, she feels well, and she denies other symptoms. She has not noticed memory, incoordination, or balance problems. Her past medical history is unremarkable, with three uncomplicated pregnancies and deliveries; she has no chronic medical conditions. Except for a mild tremor of the hands when held extended, her physical exam is unremarkable. Her neurological exam is otherwise normal. Question What lab or diagnostic study would be most appropriate for this patient? Answer Choices 1 Computed tomography of the head 2 Electromyography 3 Magnetic resonance imaging of the head 4 Serum ceruloplasmin 5 Thyroid-stimulating hormone

Answer: 5 Thyroid stimulating hormone Thyroid-stimulating hormone (TSH) would be the most appropriate test at this time. This patient presents with a history and physical consistent with benign essential (or familial) tremor. A TSH would help rule out hyperthyroidism as the cause of this patient's tremor. It is non-invasive, readily accessible, and relatively inexpensive. Unless the history suggests a more serious cause of tremor, no further testing is recommended. A CT or MRI of the head would be recommended if the patient presented with more neurologic findings on exam or a history concerning for tumor growth, such as new headaches. Both tests are quite expensive, and they are unnecessary in cases of benign essential tremor. EMG is done in evaluation of multiple nerve and muscle disorders, such as carpal tunnel syndrome, muscular dystrophy, and myasthenia gravis. Nerve conduction can be measured on EMG. EMG is not recommended or routinely done in the evaluation of essential tremor. Serum ceruloplasmin is useful in the evaluation of the relatively rare Wilson's disease. The tremor of Wilson's disease is a "wing-beating tremor" and several other findings, such as dysarthria, dystonia, splenomegaly, pallor due to anemia, and Kayser-Fleischer rings (brown to grey-green rings around the cornea), are often present. This patient's history does not suggest Wilson's disease.

A 48-year-old man presents for an annual physical exam. He has a past medical history of obesity and a 5-year history of hypertension that is currently not well-controlled. He was also recently diagnosed with type 2 diabetes mellitus (DM). He is a 20 pack-year smoker and drinks 2-3 beers per night. On exam, his BMI is 41, and BP is 145/92 mm Hg. The remainder of his exam is unremarkable. Question What is the most important matter to address with this patient concerning the prevention of morbidity and mortality? Answer Choices 1Obesity 2 Alcohol use 3 Lack of exercise 4 Hypertension 5 Cigarette smoking

Answer: 5 cigarette smoking The correct answer is cigarette smoking. Smoking is still the most important preventable cause of morbidity and mortality, and it is associated with the highest number of estimated deaths in the United States. The leading causes of death due to cigarette smoking include lung cancer, cardiovascular disease, and chronic obstructive pulmonary disease. In 2010, there were an estimated 6.3 million premature deaths in the world attributable to smoking and tobacco use. All of the other choices are causes of preventable morbidity and mortality, but they are not associated with as many deaths as cigarette smoking. Lack of physical activity is the second most important cause of preventable morbidity and mortality; it leads to weight gain and obesity. Alcohol use and hypertension related to morbidity and mortality are low in comparison to cigarette smoking, but each does pose a risk to health and should be discussed with patients.

A 50-year-old obese woman presents with severe left knee pain. She states the pain began about 8 months ago but has grown significantly worse in the last 3 months. The patient denies any trauma or event that initiated the pain. She notes stiffness in the knee first thing in the morning that lasts around 5-10 minutes. The knee pain is worsened with activity and is relieved with rest. The patient's medication list includes lisinopril 10 mg once daily for high blood pressure. She has a documented medication allergy to acetaminophen; she states this makes her break out in hives. Physical examination findings reveal a Caucasian female with a BMI of 40. There is limited range of motion of the left knee and severe crepitus. Question Considering the most likely diagnosis for this patient, what lifestyle modification would be most beneficial in relieving her symptoms? Answer Choices 1Bed rest 2 Blood pressure control 3 High impact activity 4 Screening labs 5 Weight loss

Answer: 5 weight loss This patient is most likely suffering from degenerative joint disease, also known as osteoarthritis (OA). OA is the most common form of knee arthritis. Common characteristics of OA include onset after 40 (usually in patients over 55) along with obesity; typically, these patients are genetically predisposed. Other components seen in patients with OA initially have an insidious onset of pain that will rapidly progress. Characteristics of this pain will be soon recognized as being exacerbated by activity and relieved at rest. Morning stiffness is common in patients with OA, although it will resolve within a 30 minute time period. Common symptoms patients with OA experience include buckling or giving way of that affected joint due to the bony areas impinging upon each other. Physical examination findings include limited range of motion of that affected joint and crepitus; the patient above has both. Patients may also have tenderness to palpation of the joint, joint effusion, or even palpable osteophytes. Changes that occur in the articular cartilage due to this pathology are irreversible. Conservative treatment is what should be initiated first. This should include activity modification, therapeutic exercises, weight loss, proper footwear, and avoidance of high impact activities. Use of a cane may also be mechanically advantageous for the patient, especially if it is used in the hand opposite the affected side. Knee braces or sleeves may also be utilized and provide minor subjective pain relief. Initial medication intervention of choice should be oral acetaminophen, topical capsaicin, or a traditional NSAID. It is well documented that obesity puts extra strain on the knee joints. Patients should be advised to lose 10% of their body weight in order to improve their health. It can be difficult to lose even that amount of weight. Healthcare providers should stress to patients that even minor weight loss could help people suffering from signs and symptoms due to osteoarthritis. Some studies even provide weak evidence that overweight people with knee osteoarthritis can improve their mobility and relieve pain even if they lose just 5% of their body weight within a month period. Most patients find it difficult to lose that much weight just by dieting and are more successful if they combining dieting with exercise therapy. Perseverance and motivation must be stressed. Bed rest would be inappropriate in this clinical case scenario. Screening labs is incorrect as well. Blood pressure control would be beneficial to this patient overall, but it would not directly improve osteoarthritis. The recommended therapeutic exercises for osteoarthritic patients often include muscle strengthening, flexibility, and resistance workouts.


Conjuntos de estudio relacionados

Food Safety Management Principles: Part 2 - Food Hazards and Foodborne Illness

View Set

Chapter 56: Drug Therapy for Psychotic Disorders Supplement

View Set

Software Engineering Chapter 1 & 2

View Set

Mastering Bio Ch 31 (Fungi) Dynamic Study Module

View Set

History of International Relations

View Set